#172. Orchestra, lights, beginners!

THE CORONAVIRUS AS DRESS REHEARSAL

As recently as January, the word coronavirus would have conveyed no meaning to the vast majority of the general public, whilst outside China very few, other than geographers, would ever have heard of Wuhan. All this has changed, of course, since the pandemic spread around the world in the early months of 2020.

Those of us who understand the economy as an energy system, and those people who are most concerned about environmental risk, had no reason to be any more prescient about this than anybody else.

Almost nobody saw this coming.

But energy and environmental understanding does serve to cast the current crisis into a very different light.

In short, and unless you believe in perpetual growth, the economic impact of the coronavirus pandemic can be seen as a dress rehearsal for the main event. That ‘main event’ is the onset of “de-growth”. One of the most interesting aspects of the pandemic is the light that it sheds on our ability – or, in a disturbing number of cases, our inability – to cope with fundamental change.

The energy economics perspective puts our situation into long-term context. Simply stated, the modern world was created when, in the late 1700s, the invention of the first efficient heat-engines enabled us to access the vast energy resources contained in coal, oil and natural gas. Population numbers, and the economic means of their support, have expanded exponentially since we ceased to depend entirely on the energy of food and the labour of humans and animals. This relationship, illustrated below, surely demonstrates, beyond dispute, the relationship between energy use and the quantum of population and economic activity.

Population & energy

Whether or not this relationship is understood defines the differences between two schools of thought.

For the majority of those who comment on these things, and who influence commercial and policy decisions, the economy is an entirely monetary system. Since we can create money at will, this means that there need be no limit to the scale of our economic activity (and the numbers of people which that activity sustains).

For a minority of us, though, the finite nature of the Earth and its resources implies an eventual cessation of economic and population growth. Some think that environmental considerations put limits to the scope for ‘carrying on as we are’. Others, recognising that low-cost energy is a finite resource, observe that the energy cost of energy (ECoE) is now rising in a way that is putting an end to “growth”, however much we might try to fake continuity by pouring cheap credit and cheaper money into the system.

In recent weeks, the main effort here has been to quantify, so far as is possible, the potential impact of the coronavirus crisis on economic activity and the financial system.

The detailed conclusions of these studies would probably give you far more information than you need or want to know, though the outlook for sixteen advanced economies, fourteen EM countries and the global average is illustrated here:

Prosperity trends

The bottom line is that economic activity – and the prosperity of the average person around the world – are going to be savaged by the coronavirus crisis, and that any subsequent recovery is going to be painfully slow, and incomplete. It’s by no means clear that a financial system wholly predicated on perpetual growth can survive this severe check to continuity.

This much is probably common ground with the ‘conventional’ interpretation. The difference is that, from an energy or an environmental perspective, the pandemic crisis isn’t a stand-alone incident.

It’s the first instalment of “de-growth”.

Rational responses to risk?

Members of the medical profession provide an excellent service in diagnosing our ailments and, when appropriate, prescribing treatment, but few of us would expect or want them to give economic advice. Simple courtesy suggests that we should reciprocate, confining ourselves here to economic and related issues, and leaving health matters to the experts.

It’s interesting, though, that there seems to have emerged an open rift between the British authorities and some, at least, of the experts advising them on coronavirus policy. Simply put, and with new infections continuing at a daily rate of about 8,000, some scientists think that the government is exercising insufficient caution as it lifts lockdown restrictions. It’s probable that similar debates are taking place elsewhere, though few countries seem to be as deeply enmeshed in the pandemic, or to be handling it quite as ineptly, as Britain and the United States.

Scientific interpretation is best left to the experts, and governments have other (including economic) considerations to weigh in the balance. From a lay-person’s point of view, the issue seems to be whether or not relaxation of restrictions risks triggering a serious “second wave” of infections, which could in turn force a return to lockdowns.

The operative term here is “risk”. We cannot accurately calibrate the probability of a second wave, but we can reach a pretty effective estimation of the consequences should it happen.

The subsidiary question is whether there are “right” and “wrong” – “prudent” or “irresponsible” – ways of emerging from lockdown.

It’s almost impossible to overstate the economic implications of a second wave. China aside, the coronavirus struck most countries’ economies in late March, so first quarter output was only reduced by about 3-5%. In a second quarter wholly overshadowed by the pandemic, activity is likely to have fallen by between 40% and 50%.

A cautious, incremental approach might see this year-on-year gap narrowed to perhaps -30% by the fourth quarter, with something close to normality being restored by the end of 2021. This might only be “close to” normal, because there are some sectors which it would be imprudent to reopen until the virus risk is very largely behind us.

Unduly rapid exit, on the other hand, risks triggering a second wave of infections, at which point economies would be forced back into lockdown.

Any ‘lockdown 2.0’ would be far worse than the original one. It would probably have to last a lot longer than the first version. As well as forcing economic activity sharply back downwards, this would strip people of much of the hope that has sustained them through the period of restriction. It would throw government and commercial planning into disarray, and would risk both severing supply lines and triggering a full-blown financial crash.

Any recovery thereafter would be very gradual indeed, and might take too long to avoid permanent, perhaps even existential, economic and financial damage.

Issues of responsibility

It cannot be emphasised too strongly that no encroachment on the preserves of the medics is intended here. The world already has more than enough ‘instant experts’ on the coronavirus, and certainly doesn’t need any more.

The aim is simply to examine the possible economic consequences of allowing the system to risk being hit by a second wave of infections. The implication, though, is that purely economic probabilities favour caution.

Of course, it can be objected – and quite correctly – that official consideration needs to be given to matters that are neither medical nor economic. Lockdowns restrict freedoms, are stressful, and have extremely painful human consequences, including physical (though not, strictly speaking, social) isolation from relatives and friends. Nobody wants to stay in lockdown any longer than is necessary.

This doesn’t mean, though, that exit strategies can’t be prudent, and nuanced to remove the worst human and economic consequences whilst also minimising the risk of a second wave. It seems logical that the authorities could decide what should, and what should not, be reopened, on the combined basis of importance, and of comparative safety. If people can work, or meet, at safe distances, there seems no reason for stopping them from doing so. Cramming people onto beaches or into aircraft seems far less advisable.

This discussion has probably reached – or passed – the point at which some readers riposte that the coronavirus ‘is no worse than flu’, ‘only affects the elderly’ and ‘leaves no lasting health impairments’ (though each of these points seems unproven). Others might reference ‘herd immunity’ (although, even in badly-hit England, official survey data indicates that only 6.78% of the public have antibodies).

These are opinions, to which anyone is entitled. But the problem with such arguments is that none of us makes decisions for himself or herself alone. We might, as individuals, think that risk is low, so we’re relaxed about crowded spaces, and pay little attention to precautionary guidelines. It can be argued that we have a right to make that choice, always presupposing that we accept the risk that we might be wrong.

But the risks of such decisions are not confined to those who take them. During the Second World War, night-time blackouts were imposed, to make it harder for enemy bombers to find their targets. This would have been pointless if even a small minority, disagreeing with the blackout policy, had kept their homes lit up like Christmas trees.

At issue here is collective responsibility, and the question of adhering to rules with which we, as individuals, might disagree.

The intelligence factor

The merits or demerits of rapid or cautious “exit strategies” from lockdown are not intended to be the main focus of discussion here.

Rather, the issue of greatest significance is the way in which, collectively, we have responded to this ‘dress rehearsal’ for de-growth.

The view expressed here is that de-growth has become very probable indeed. For purposes of explanation – and with a new downloadable summary of surplus energy economics in preparation – it might suffice to note that all economic activity is a function of energy, and that the energy cost of energy (ECoE) determines how much of any accessed energy is consumed in the access process, and how much remains for all economic purposes other than the supply of energy itself. Needless to say, no tinkering with the financial system of ‘claims’ on economic output can change the fundamental energy (not financial) dynamic which determines our prosperity.

Analysis of these trends indicates that de-growth had already started, well before the economy was hit by the pandemic. During 2018-19, sales of everything from cars and smartphones to chips and components had turned down. Unmistakable signs of stress were already starting to appear right across the financial system.

The arrival of de-growth finds us with a financial system that has been rendered unnecessarily fragile by futile efforts to counter “secular stagnation” – and, latterly, de-growth – with monetary gimmickry. Not content with allowing escalating debt to create cosmetic activity and “growth”, the authorities had already resorted to monetary policies which, as well as paying people and businesses to borrow, had destroyed returns on invested capital, with particularly adverse consequences for pensions.  The following charts illustrate the extent of financial exposure.

GDP & obligations

You can take your pick between escalating ECoEs and worsening environmental risk as the primary drivers, but the onset of de-growth looks inescapable.

This, simply put, poses a challenge unprecedented since the start of the Industrial Age. There have always been recessions, of course, and depressions have occurred at longer intervals. But these events, however severe, have never amounted to a permanent cessation and reversal of economic growth.

Another way to state the case is that de-growth has put an end to ‘business as usual’. Have we the intelligence, individually and collectively, to adapt to this drastic change? Moreover, do our societies and our institutions have the systemic intelligence to respond rationally?

This isn’t the place to revisit what de-growth is likely to mean, but we can expect fundamental change in economic, political and other areas. Economically, products and services are likely to be simplified, with the same happening to supply processes (as part of a wider trend towards unwinding the complexity created during more than two centuries of growth). Whole sub-sectors are likely to be de-layered out of existence. Any culture in which people derive their sense of self-worth from material affluence is likely to be undermined. Current distributions of income and wealth might not be tenable in a shrinking economy.

It remains to be discovered whether we have the intelligence (which is not the same thing as cleverness) to adapt ourselves to such fundamental changes.

Seen as a dress rehearsal for de-growth, the coronavirus crisis gives us scant reason to trust that “it’ll be alright on the night”.

 

 

125 thoughts on “#172. Orchestra, lights, beginners!

  1. Hi Tim,
    Another great post so thanks for that. I love your measured approach and tone.
    As a dress rehearsal for the real show things do not look good from where I am sitting. Here in Ireland the FinMin has stated its full steam ahead with economic growth policies to pay for everything.So no change there. How many steps down (second wave?) do we go through before anything changes. I know you do not know the answer and I am not really expecting one as no one knows. We live in interesting times indeed.
    Thanks again, Richard

  2. Thanks Tim, for this new addition to our discussions.

    You end by suggesting that, “Seen as a dress rehearsal for de-growth, the coronavirus crisis gives us scant reason to trust that “it’ll be alright on the night”.

    Given the above, I wonder if you have in mind an idea how you think things will be playing out, at least for the UK, in say six month, a year, three years ten years etc?

    • I’m increasingly worried about the prospects for the UK. This situation is appallingly dangerous.

      As I see it, no economy can really afford the prospect of a second wave, and the second lockdown that would be inescapable if that happened (because millions would isolate out of fear, even if they weren’t told to). These are issues that call for individual and collective responsibility.

      So, watching a minority (but a sizeable one) crowding onto beaches (etc), with seemingly complete disregard for others (and to the dismay of the locals), I’m not sure who is the more irresponsible – those involved, or the government which failed to think this through.

      If other countries handle this better, but the UK gets mired in a destructive second wave – or a long-drawn-out limbo between lockdown and reopening – then the country is in very, very big trouble. Lockdown 1.0 is already creating enormous fiscal pressures, and 2.0 could cause a full-blown Sterling crisis. For any country so dependent on imports, this would be a disaster in the making.

  3. Re:
    ” Have we the intelligence, individually and collectively, to adapt to this drastic change? Moreover, do our societies and our institutions have the systemic intelligence to respond rationally?…It remains to be discovered whether we have the intelligence (which is not the same thing as cleverness) to adapt ourselves to such fundamental changes.”

    Small communities have the best chances to adapt. As social mammals, the clan/tribe/small town has first hand knowledge of each member, and takes the responsibility to deal with deviants. This evolved over hundreds of thousands of years as it did with other social mammals.

    Intelligence of individuals varies. In groups of 100-200, things can average out somewhat, barring inbreeding problems which can degrade the group’s intelligence. Strong cultural heritages (usually religious) can alter communal intelligence both up and down.

    Individual free will is poorly understood, and depends upon precise definition. I am amongst the skeptics. But the community has primary responsibility, so let’s go with that.

    I highly recommend a short compilation of lectures by Ronald Wright which addresses similar predicaments in prior civilizations:
    https://www.cbc.ca/radio/ideas/the-2004-cbc-massey-lectures-a-short-history-of-progress-1.2946872

  4. Dr. Morgan

    I greatly appreciate your attempts to outline how our economic future might broadly unfold, and also the caution and reserve you show when doing so. Perhaps because I am too caught up in the current news cycle, but here in the U.S. today least it feels like we have entered a period when the course of social and political events will be very hard for anyone to predict.

    What political or social shifts will occur in the U.S. from the recent protest-to-riots-to-looting events? I have no idea of how the reaction/counter reactions will play out. But has to wonder if the broader deeper issues are about declining prosperity.

    I wonder too if our social response to the coronavirus might change over time, as economic buffers are depleted, and we face to the choice between the risks of going to work versus the risks of not having enough food. IF younger people or much less at risk from this particular virus, might they embrace the risks and put aside their masks and demand to work, even in the midst of a second wave? (A rhetorical question). Only time will tell.

    • Thanks. I’m appalled, of course, by the riots and by their cause. The outrage seems fully justified, though I also wonder about the role that economic hardship might have played in creating a ‘bonfire awaiting a spark’.

      I hope this doesn’t sound too glib, but I do wonder about societies (Britain being another) which glorify wealth and ‘celebrity’ whilst perhaps doing insufficient to help the weakest, and sustain social cohesion. This crisis seems to be reinforcing the case for a mixed economy balancing the best of the private and public sectors.

      My case-study numbers on this show the average American about 20% poorer this year than he/she was in 2019, with not much of a recovery in prosperity in 2021. This seems bound to push large numbers of the poor and the not-so-poor into extremities of hardship and despair.

      This ZH item seems pertinent:

      https://www.zerohedge.com/political/epitome-rioting-irony-and-ignorance-two-tweets

  5. Simplification. Yes. And it was only in the 1970s that The Two Ronnies did their sketch where Ronnie Corbett asked for an ice cream, the Ronnie Barker the ice cream man asked him what he wanted from a whole list of options that drove Ronnie Corbett up the wall. Brands, brands, brands, And choices and variations and options.

    • If times are hard, it’s easier to manufacture and distribute, say, ten kinds of biscuit rather than fifty.

      As a child, I spent a lot of time in a very remote place where the choice of biscuits at the local grocer’s was just two – “plain or fancy?” Did that make the local people feel miserable and deprived? I rather doubt it.

      It needn’t go quite that far, but having less choice of biscuits might be better than a shortage of biscuits.

  6. Your analogy of the blackout is not very good, because switched-on lights are visible to the human eye, whereas germs and viruses and bacteria never are. If we go down that route, why shouldn’t I claim that you potentially gave that rotten cold that lasted three days last year and sue you, telling you that it’s for you to prove that you WEREN’T to blame?

    • With respect, the point I was making was that blackouts were only effective if implemented by all, including those who might disagree. In that sense, bomber pilots could have exploited a weakness created by a lack of collective consistency, and the virus can do much the same. If somebody ignores distancing, crowds onto a beach (say) and contracts coronavirus, it’s not just him or her at risk, but anyone with whom he or she is then in contact.

  7. A pest controller once explained to me how they judged carrying capacity on a given site, giving an example with rats. When they laid out all killed, they looked at the demographic profile and the more adults, the more the population was straining against natural limiting factors, while the more young and the younger the spectrum overall, you had a growing population so resources were still available. When population pressure was intense, competition, up to cannibalism even, resulted in the most extreme cases with most individuals being adult with the strong and males advantaged.

    Globally, a record number of countries now have birthrates below current population replacement levels, including nominally wealthy ‘developed’ ones, which indicates a lack of confidence in the future. This trend is not new and seems to be accelerating, while the negative drivers are only gaining in strength if anything. Downwardly-mobile people are simply struggling to afford to breed.

    As for staged collapse, the US always leads the UK given the culture whereby they are more prepared to accept savage injustice for their fellow-citizens, so as the pain here also starts to lap at the ankles of the employed, a threshold will be reached after which indicators of social breakdown will show. With respect to the current govt.’s ability to handle the fireworks, we don’t have to guess, we can see it everyday now with the viral debacle. It is quite hard to be optimistic at the moment.

    • It is indeed hard to avoid pessimism, even though some countries are handling this a lot better than the US and the UK.

      This seems to be a point where the wealthy – and the reasonably wealthy – need to ask themselves what kind of society they want to live in.

  8. Thanks Tim for this – as always succinct and clear.

    I wonder how much we can learn from historical depressions as a model for degrowth. Recessions in recent years tend to be fairly short. If one goes back to the late 19th century or the post-Napoleonic war era the UK experienced economic declines lasting decades. I imagine that this has been a fertile area for economic historians but how much is known about the social changes and impact on the ordinary person? How did they adapt? What did they do?

    • Thanks.

      Britain in 1815 was all but bankrupt, having been at war since 1792. Perhaps foolishly, the government had promised to repeal income tax as soon as the war was over, and MPs made them do it. Unrest was rife (Peterloo is just one example).

      But there were two saving graces. The first was that the Industrial Revolution was under way, and the second was self-sufficiency in food and other essentials.

      In the inter-war years, Britain probably wasn’t too far from revolution, as happened in many other European countries. Hardship was rife, inequality was extreme, and millions of men, now unemployed, had been trained to use guns (of whom many had brought weapons back with them). One interpretation is that the unions could even have taken over in 1926, but knew they had no solutions to offer.

      We can cope, I believe, with almost any amount of hardship so long as, eventually, it ends, and we feel that we’re “all in it together”. De-growth isn’t like that. We need to change our thinking, including our expectations, which certainly shouldn’t be impossible, but will be difficult.

  9. @Dr. Morgan
    Thanks for this article..which I plan to put to some (hopefully) good uses. One of the points I am trying to make to people who think that the current problems in the US are all ‘racial’ in origin is that we need to look carefully at how successful biological systems work…because, in the end, we are just another biological system. It’s easier to be objective when observing fruit flies or orangutans.

    Every living creature needs to be able to distinguish allies, enemies, and bystanders. From the Covid-19 perspective, the world is awash in genetic material which is potentially harmful to humans. But we have some allies in the gut microbes, who are a dominant factor in our immune system which keeps the harmful guys at bay. But the gut microbes can only do their job if we give them the nutrition that they need to thrive, and the stable environment they need. Sadly, most Americans have either neglected or mistreated their microbial partners, and a huge number now suffer the consequences…including death from Covid-19.

    A number of years ago Spike Lee (a black filmmaker) and Samuel L. Jackson (a black film actor) were discussing the matters of the day and rejected the notion of doing something ‘to help our people’. They snorted at the idea. Which tells me that they were smart enough to see that some people who have black skin are their enemies. I hope that there were, in their lives, some people with white skins who were their allies. In short, what we need to be focusing on is allies and enemies…not skin color.

    What is required of an ally? I suggest that it boils down to:
    *access to energy
    *information about how to flourish in a hostile environment
    *someone who needs what I can give
    To give three simple examples:
    *Someone who knows how to grow a garden with minimal inputs.
    *Someone who knows how to retrofit a house to make it mostly passive in terms of heating and cooling
    *Someone who is a skilled midwife

    I suggest that those who are trying to manipulate us will try to focus on making racial differences and nationality and perhaps things such as sexual orientation and class as the decisive factors. Lee and Jackson were too smart for that. The rest of us should take note.

    Don Stewart

    • ‘ I hope that there were, in their lives, some people with white skins who were their allies. In short, what we need to be focusing on is allies and enemies…not skin color.’

      Idealistic people have been trying to build ‘multicultural’ societies since the founding of Babylon. And they have been failing for just as long, leaving human wreckage and failed civilisation in their wake. Sumaria fell after their lands were colonised by Semites. Babylon fell because it tried to be home to all of the varied peoples of the world. But when the Persians invaded, too few of those multicultural citizens were prepared to lay down their lives for Babylon. A nation that had previously dominated fertile crescent, could not muster the strength to defend it’s own walls. Any serious civilisation is basically mono-racial. No amount of wishful and idealistic thinking will change the ingrained nature of human beings. You cannot undo millions of years of evolutionary survival tactics with indoctrination, censorship and political ideology.

      Idealists like to pretend that they can ignore history, because this time will be different, because of x or y. How many times does a bad idea need to fail before an idealist will drop it? An infinite number, apparently. When a beloved idea fails, idealists always assume that it failed because of the evil racists, or the evil western capitalists, or whatever other group of real or imaginary people seem to stand in their way. They will never accept the fact that the world doesn’t work the way they think it should and will treat failure as a sign that they need to double down, with more control, more censorship, more punishment; in an attempt to force human society into the mould that they insist it must form to.

    • Tony:

      I come at this differently. I believe that all people are of equal value, irrespective of race. We’re not, then, ‘white British’, or ‘black British’, but simply ‘British’. That’s the ideal to aim for.

      ‘Multiculturalism’ doesn’t further this ideal. It puts people into groups or ‘communities’, thus emphasising and entrenching differences.

    • The US used to be called a melting pot. Canada opted for “multiculturalism.” The melting pot has begun to fail, with “woke” progressives pushing identity politics, and the far right supporting white supremacy. (both are against melting pot)

    • Too much idealism and wishful thinking here from people that want human beings to be different things to what they really are. It makes no difference how much you want human beings to accept the idea of a multi-racial utopia. They are hard wired to see people of different racial groups as foreigners. In much the same way, red and grey squirrels do not recognise each other as kin.

      The idea of a melting pot worked reasonably well for as long as the US remained essentially a pan-European milieu. Those people were similar enough racially and culturally to be able to put aside differences and form a common nation based upon European and specifically Anglo-Saxon individualistic idealism as embodied in the US constitution. It helped as well that all European peoples essentially share the same Christian religion, with only minor differences between catholic and protestant denominations. Both live by the new testament and accept the same basic cannon, with an ethical code based upon the morality of the old testament.

      The melting pot failed when European America attempted to integrate people that were non-European; and when a broadly Protestant Christian America had to reconcile with other contradictory religions, like Islam. These peoples are simply too different to integrate into the White European milieu and have essentially formed nations within a nation. The riots gripping the US are the result of irreconcilable differences between white Americans and the dominant culture that they built and non-white America, which wants something entirely different. There is no melting pot between the two. I suspect that the US is heading towards another civil war.

    • Multi-racial is an ideal, of course, but it seems to me that having ideals is better than not having them. It depends on integration enabled by equality of treatment and of opportunity.

      Multi-cultural, on the other hand, has come to mean dividing societies into different ‘communities’, defined by race. As such, it’s an obstacle to integration.

    • This much is true. Race is a meta-ideology and race ideologues tend not to respect different racial ideological points of view. The cult/ideology/religion of race is not based on science or rationality but based on imagined ideological differences that are abstracted from the melanin levels in our skin.

      To turn diverse environmental adaptions into discreet ideological frameworks is simply another form of sociopathy which seeks to dehumanise and commodify human beings into black cultural goods and white cultural goods.

      However, whilst racialised thinking persists then the true nature of the conflicts over scarce resources, scarce decent jobs, scarce decent housing, scarce land availability remains obscured in favour of melanin levels in our skin.

      As such, People should be coming together to resolve our many paradoxes, not being encouraged to fight over them on the basis of melanin levels in our skin.

      Disintegration, fragmentation, segregation, balkanisation are the products of self-hate that is inculcated by divisive race ideologues who thrive on the projection of their self-hate. These sociopaths in turn exploit our democratic freedoms to proselytise self-hate.

      This self-hate and the projection of self-hate produces human/societal/community infighting which is dramatically reducing social productivity at a point in our history when we need the highest levels of social productivity imaginable in order to co-create a sustainable, sufficient and resilient future together.

      When anyone rejects the need for integration whether through cynicism, skepticism or hate, then they are actively seeking to simultaneously reduce social productivity.

      Social productivity is enhanced by the integrative effects of wisdom and compassion with both requiring collective knowledge, collective understanding and collective empathy.

      Cynicism and skepticism however thrives on a lack of collective wisdom and collective compassion and thereby seeks to limit the possibilities of human agency, limit the possibilities of human ingenuity and limit the possibilities of human adaptation.

      We need to be thinking collectively, not individually on the basis of melanin levels in our skin.

    • Thanks Steve, well put.

      A lot of items are appearing in the ‘liberal’ media about what the virus crisis means for X, Y or Z ethnic or social group, and I’ve no problem with that.

      But there’s a glaring lack of coverage of what the crisis means for the poor, irrespective of race.

      In the US, long-standing racism seems to have made black Americans poorer than white Americans. If so, then what we’re seeing isn’t a simple straight line through race to unrest, but a line that runs from race through poverty. Poverty, at least, is something that we should be able to tackle.

      I often wonder why poverty isn’t a bigger issue in politics and the media.

    • There is a great line in Michael Powell’s movie I Know Where I’m Going, starring the wonderful actress Wendy Hiller. I think the line is delivered by Pamela Brown. The plot involves a smart London girl who is going to marry Imperial Chemical Industries…or at least the general manager of that company, on his private island off the coast of Scotland. So Wendy takes the train to Scotland, but due to a storm, can’t get across to the island. Which leads to the real plot which is meeting the now impoverished Laird of the island, who is on a short leave from WWII. Wendy remarks on the poverty of the Scots. Pamela says ‘we aren’t living in poverty, we just don’t have any money’.

      Society’s inability to come to terms with that distinction is, IMHO, a principal reason we cannot come to grips with poverty in the US and Britain.

      Don Stewart

    • Thanks Tim and Don.

      Part of the problem in my opinion is universalistic liberalism which idealises a society of pure equality. The question is then equalise to what. If we equalise to working class standards of living (what I term “levelling down” then in theory that would probably work out fine in relation to wealth and resource distribution.

      However, the proponents of universalistic liberalism of which “critical theories” are a part, is usually promoted by the middle class ‘liberal’ intelligentsia who, far from wishing levelling down, want equalisation to occur at their level of wealth and resource consumption, which as we know would require around 3.5 planets.

      Far from acknowledging that, the ‘liberal’ intelligentsia instead focus (or should I say project) their resistance to levelling down on to the upper classes of the Davos elite with the idea that the top 10% earners should be paying even more than the 60% of tax receipts that they are already paying. Whilst the middle classes pay only around 30%.
      https://www.ifs.org.uk/publications/9178

      On the back of this, the liberal establishment feels justified in starting a race war in America, a Brexit war in the UK and god knows what else they have planned in the coming years.

      Essentially then, at least from my perspective, the main basis of the politico-cultural war is taxation with the middle classes protecting their distinguishing privilages of rank, office and qualifications from levelling down and then proceeding to project on to the 10% the need to reallocate their incomes and in the process destroy the investment base of the country.

      As such I am now of the conviction that the Liberal Establishment with the support of the Liberal middle classes wish to destroy society and from the ruins they hope to prosper by blaming everyone but themselves. In this respect, one could say they are trying to activate a revolution with no clear plan except tax the rich and create a socialist society with them in charge.

      This might sound all worth it from their narrow interest class point of view considering that many people and their families, their businesses and properties are going to suffer extreme harm, loss or damage in the process, which highlights why this is all happening now under ‘Conservative’ governments.

      However, what I and many millions of other working class people fear is the ideological nature of their revolution. So we have Race Ideologues, Trans Ideologues, Socialist Ideologues and god knows what other Ideologues wishing to impose universalistic liberalism on every person in the Western Hemisphere all because they resist levelling down and paying more taxes for THEIR universalistic ideals.

      So yes, misplaced idealism generally does end up in ruin. Not because of the idealism in itself but because Ideologues (sociopaths) can’t do what it takes (sacrifice) to realise their own ideals.

    • The fundamentals were once described as a struggle “between those who have, to keep what they have got; and those who have not, to get it”. (Robert Lowe, I think).

    • I agree, Tony. Steve G.’s post and Tim’s reply are idealistic. It would be nice, but… Much fear and distrust of strangers is learned, from a young age. But there is an innate caution. That developed during natural selection over tens of thousands of years. Those who were too trusting were most vulnerable to being killed. They got Darwin Awards, as lifespans didn’t go beyond breeding ages..

      Massive population overshoot exacerbates distrust and conflict. Recall the value of neighbors in pioneering days and in sparsely settled areas. Rarity increases value; overabundance devalues.

    • @TonyH,

      I come at this differently too. You seem to be projecting your experience of being a human as universal in all aspects, onto the rest of us. I suggest you don’t and never can, and it is hubris to suggest otherwise.

      For example, you blame the collapse of civilisations on “idealistic” people who “will never accept the fact that the world doesn’t work the way they think it should” and that “they are hard wired to see people of different racial groups as foreigners”.

      But this is a straw man argument, because surely you can never truly ‘know’ the inner experience of all the other 7.8 billion of us?

      If you do, then where does your authority come from? How can you be so sure to speak and define us in the terms you so vividly describe in your posts here, suggesting that ALL of us are by design ultimately and unalterably dominated by violence?

      Another example is you mistake correlation with causation vastly over simplifying a complex multi-layered process to “Sumaria fell after their lands were colonised by Semites. Babylon fell because it tried to be home to all of the varied peoples of the world.”

      What does “fall” even mean, if the highest principal is violence, but the aim of your world view presumably would have been to have Sumaria and Babylon preserved rather than fallen? There is a deep contradiction here.

      Perhaps instead, some of the essential defining characteristics that drive humans to create civilisations are empathy and creativity that have self-evidently out-competed competition itself? Or there would be no Babylon or Sumerian or any other cultural remains to pick over the bones of.

      Whilst we try to build a viable long lived civilisation, perhaps also there are ‘natural monopolies’ best administered in the public domain of empathy than private one of violence?

      Or as Steve Gwynne suggests, lets not “turn diverse environmental adaptions into discreet ideological frameworks” because they are “simply another form of sociopathy which seeks to dehumanise and commodify human beings […] We need to be thinking collectively, not individually.”

    • @Natasha
      Sounds like an overreaction to a presentation of the historical record. Wars have been with homo superstitious forever. And, despite Pinker and other idealists, as humans go further into overshoot, conflicts over shrinking natural wealth like fertile soil, clean water, fossil fuels, other mineral deposits, etc. conflicts will increase.

      I continue to offer to wager for charity on longbets DOT org on this. That site is a subsidiary of The Long Now Foundation, started by Kevin Kelly (Wired Mag.) and Stewart Brand (Whole Earth Catalog)

      I’ve posted links here about the Maximum Power Principle, which includes humans with all life forms in seeking energy gradients to sustain growth and replication. Ecological systems scholars understand this, and Energy Economics is consistent with MPP. (Biophysical is the popular term now) Cooperation is required by social mammals. That is not in question. But groups compete with other groups as well. If you want the MPP link, email me:

      kurtzs AT ncf DOT ca

    • Thanks Natasha and Steven

      The maximum power principle when applied to social productivity implies cooperation not competition, with the latter wasting energy and the former aggregating energy.

      Try an experiment at home. Light two candles and see which produces the biggest flame by area. Two single candles or two joined together as one.

      In this respect, the conflict between the Liberal Establishment and the Conservative Establishment does not produce maximum power but minimal power.

      The whole point of the dialectic is to transcend the thesis and the antithesis into the synthesis.

      Perhaps civilisations collapse because humans unerringly don’t follow the maximum power principle. In other words, they resist the forces of Nature by not cooperating according to universally accepted values and rules.

      The whole is greater than the sum of its part.

    • @ Steve G.
      My understanding:

      MPP applies to life forms – including plants, bacteria, fungi, insects, reptiles, fish, birds, and mammals. All seek to maximize energy throughput, with the only exceptions I’ve thought of being in a small% of humans via voluntary simplicity and suicides.

      It applies to groups of social mammals – tribes, clans, small villages. I agree that cooperation is most important, even amongst various tribes in a region. MPP is not about competition vs cooperation. It is about maximizing the thriving, reproduction, and endurance of the group. Cooperation is paramount for this unsocial mammals and many other species. But they will compete for resources when necessary.

    • typo (auto complete)

      “unsocial mammals” should be “in social mammals”

  10. Tim,

    Worth pointing out – you may have done it in the past but the Arab Spring was to a large extent fuelled by economic hardship, partly of course brought about by climate change.

    There is also this, 54 million US citizens at risk of going hungry

    https://www.theguardian.com/world/2020/may/31/people-are-going-to-go-hungry-pandemic-effects-could-leave-54m-americans-without-food

    Even if 10% of them start starving the results might well be ‘interesting’

  11. Dr. Tim, thank you for another insightful article. It is always a pleasure to read your work.
    I understand you’re trying to frame the debate here, and although I learn a great deal from other contributors here on their pet issues, I will try to keep my comments within your framework.
    I follow the German press on covid-19 more than I do the British press.
    The German press is much more objective and the British press is just constant hysteria.
    Here is a link to ZH that maybe of interest to some. It speaks for itself, and my views align with much of what is written.
    https://www.zerohedge.com/markets/german-official-leaks-report-denouncing-covid-19-global-false-alarm
    We do need to look at the post Covid-19 world. Your own figures show for example how the UK has been in decline (ca. 0.8%pa) since 2003. So people are beginning to feel it pinch them.
    You also point out the notable decline in High street sales, smartphones etc.
    This year 2020 will be “The Year of No Holiday !” for most people.
    Unemployed pilots and flight attendants will join the dole queues, along with duty free sales assistants, bar-staff, waitresses and cinema staff.
    Make no mistake, this is a big hit to the economy. BAU is not coming back and although we might see the occasional uptick, the general direction is a slide to a deep depression.
    The UK is so dependent on Services, yet many of these jobs are never going to return.
    We have not even factored in Brexit yet. We saw what happened to Ukraine when they turned their back on their biggest trading partner back in 2014.
    Sure, the Germans want to keep selling cars to the UK, but what are the British going to pay them with ? They have flogged all their assets as it is.
    With every nation having their own set of problems to deal with, the USA, Saudi, Europe, etc. I do not see anybody coming to the UK’s aid should the GBP come under significant pressure.
    This is a scenario which I am fearful may be just around the next corner.

    • Thanks Johan.

      As you appreciate, my aim here was to step outside the mainstream debate and ask whether we are intelligent (perhaps ‘rational’) enough to solve really big problems. The conclusion seems to be that we are not.

      I’m not suggesting we are or should be “Thinking Machines“, like the character created by Jacques Futrelle, but simply rational enough to weigh near term desires against slightly longer term but much larger risks.

      A second wave looks to me a good test of this question. If we were wholly rational, we’d probably accept some extended privation now to avoid something potentially far worse a bit later on. I’ve run some numbers – admittedly theoretical – on what a second wave would look like economically, and ‘horrific’ just about describes it.

      I do try to keep abreast of developments in Germany, which is one of the 30 countries on SEEDS. Interestingly, German prosperity per capita has carried on rising until 2018, albeit only very gradually, with taxation taking an ever larger slice.

      The UK situation looks precarious in the extreme. This might justify an article here, perhaps.

  12. Good article, Tim! As usual, good writing and excellent framing.
    Here in America we’re conducting a fine experiment for the ROW right now, by having lots of people mix freely and get up close and personal in the streets of our large cities. Not saying it’s uncalled for, but it’s happening and we’ll know if there’s going to be a second wave in the U.S. in about two weeks.

    In the above comments you say, “This seems to be a point where the wealthy – and the reasonably wealthy – need to ask themselves what kind of society they want to live in.”

    Unfortunately, that is not how rich people think. They think that the whole point of being rich is, precisely, to exempt oneself from consequences. They don’t live in a society, they are members of a free-floating exclusive social club that can go anywhere. Who cares if America goes under, they will fly to their retreats in New Zealand.

    As usual, art, even popular art, susses out the prevailing ethos and illustrates its logical outcome. Two recent pieces of fiction illustrate the mindset perfectly – The Hunger Games, where 13 outlying provinces living in near poverty support a single, shining city, and the environmentally dystopic Matt Damon film, Elysium, where the mass of people on earth living in abject poverty and a toxic landscape export the goodies to a ringworld around the planet, which is a rich person’s beautiful perfectly isolated bubble world – Elysium, in short.

    I recommend listening to the Chris Martenson interview of “Fourth Turning” author, Neil Howe that someone linked to recently. Really interesting, In it, Howe recommends reading the book “The Great Leveler” by Walter Scheidel. Scheidel argues that there are only 4 things in history that reverse extreme inequality – total war, total revolution, famine, and plague. “Reform” never ever cuts it. Discussion of this begins around the 29:00 mark.

    • Thanks Tagio

      I’m watching what’s unfolding in US cities with something near despair. I understand the anger, and just wish there was some other, better way of getting understandable grievances addressed. People have hoped for that since Dr King and Bobby Kennedy, I guess.

      These days, the rich aren’t those with huge pots of cash, but stockholdings in some very big corporations, and you can’t pack those stockholdings into a suitcase and take them to NZ. So their own best interests seem to lie in keeping society working, and their businesses making money, rather than letting things go to heck and taking the first plane out. Maybe I’m being too rational, though….

      I saw an entertaining piece on ZH about a journalist, urging rioters to burn down everything, until suddenly his nearby Starbucks was trashed and his own gated community was under threat, when his stance changed, the rioters becoming “animals”. A great read.

    • It was the same in the U.K. in 2011. The middle and upper middle didn’t care a jot about riots in Hackney, Lewisham etc. until it started leaking out into their domains. I remember the story of a restaurant in Battersea suddenly being subjected to a raid and the rich (sic) punters being forced to hand over everything they had on them.

      Once that started happening there were suddenly police everywhere cracking down on any disturbance. It was finished in two to three days after that

    • I wrote a controversial paper about that when I was at TP, calling 2011 the first ‘consumerist riots’.

      Message #1 from the system – “you’re inadequate if you don’t own X, Y and Z”.

      Message #2 – “sorry, you can’t afford them”.

  13. Tim.

    Like another commentator above, I’m intrigued by the predicted declines in birth rates and eventually population as we all turn Japanese. The following ZH article focuses on the decline of birth age women, but births per woman are also showing accelerating decline as we all turn Japanese. I’d regarded these trends as a function of wealth, access to education and career choices for women rather than increased poverty since the trend seems strongest in the richer parts of the planet and predates by several decades our current position.

    How do you see demographics affecting our energy position with fewer people arriving and those who are here aging significantly. Older people, retired people, consume less? Travel less? Have a lower energy footprint?

    Will the effects cancel each other out and age adjusted energy consumption per head will be more stable?

    https://www.zerohedge.com/geopolitical/global-births-population-potential-mothers-down-down-down

    • Human physical labour is a very small part of total energy consumption, and our main role is the direction of energy from other sources. This doesn’t mean that physical tasks are unimportant, but it does mean that we don’t need huge numbers of people doing physical work for the supply side of the economy to function. If we overstate the role of human physical labour, we can become unduly worried about an ageing demographic. We don’t need a huge cohort of young people performing physical tasks to support a growing number of retirees.

      As you’ll see in the first chart here, population numbers have exploded in tandem with energy use, and I’m not alone in thinking that our numbers are now a long way into overshoot. Rightly or wrongly, I’ve long put a lot of faith in education and female empowerment to address this.

      The prosperity connection seems capable of cutting both ways. Poverty might limit birth rates, but there’s some evidence that affluent people delay having families, and have fewer children.

  14. Setting aside for now how much of a threat the coronacrisis is and therefore what the best response would be, most regimes around the world have seized the opportunity to get whatever they want under the cover of ‘looking after their people’. This is more than matched by any entity with influence, making power grabs commensurate with their strength, like corporate appropriation of public space, facilities, institutions and serious roll-backs of basic citizen’s rights in all directions.

    By the time the average person in the street realises, it will be too late to meaningfully respond and the masses will be newly powerless as well as broke after their savings and incomes collapse simultaneously while we’re sedating ourselves on pay-per view TV and at least the legal drugs. Bread and circuses, nothing really changes and the illusion of freedom we believe is evaporating. I bet deaths of despair (sh*t life syndrome) will end up dwarfing those attributed to the coronavirus.

    I can’t understand why if there is a second wave possibility we haven’t yet seen it in the poorest countries where teeming multitudes can’t afford to distance, stop normal activities through which they earn the food to stay alive until the next day, access healthcare or exercise basic hygiene.

    • I’m not sure I can provide any encouragement, except that minority regimes down the ages have thought they can keep ‘the masses’ in place through force, money and intelligence (for which read technology in the modern setting). The Tsars, for instance, had wealth, huge numbers of soldiers and a formidable secret police force but still got overthrown.

      The issue about a second wave isn’t probability alone, but probability x scale of consequence. This said, it’s indeed encouraging that some poorer countries haven’t had a second wave, but patterns of activity are different – few citizens of poor countries flock to beaches, and few of them demand the right to fly to and from China, etc..

      As this is a virus transmitted person-to-person, this may be a pandemic of mobility, and therefore worse for more affluent, more mobile societies. I fear for what this crisis could mean for the poorest countries.

  15. Tim,
    Given the distinction you’ve drawn before between the real and financial economy, to what extent is Covid merely accelerating the collapse in the financial economy that we could expect anyway with a much lower impact on the real economy.

    I don’t underestimate the default risk for the debt mountain precariously balanced on the financial economy but my anecdotal observation from inside a manufacturing business is that the real economy is taking a much lower hit.

    • Thanks Alan.

      The answer is yes. Our financial system has grown out of all proportion to the real economy (a problem made worse by a widespread failure to understand what the ‘real’ economy is and how it functions). It was always a matter of time before this corrected, and was never going to take much of a jolt to trigger it. Pre-virus, I’d been looking at China, which has doubled its GDP whilst quadrupling its debt, and where very serious signs of financial stress had been increasingly visible over the last two years – defaults (including SOEs), failure of non-bank lending platformsetc..

    • Whats the figure

      World trade in goods & Services is about $26 trillion pa, turnover in the foreign exchange markets is $6tn/day……………..

      The bulk of the global economy no more edifying than the local bookies (Betting shop for non UK people)

  16. Hi Tim, all.

    I think a possible solution to our growth dilemma is waiting in the wings.

    Bearing in mind that neoliberalism (the trend towards hyper capital efficiencies via privatisation, market access and economies of scale) in conjunction with globalisation (the meta platform from which to facilitate neoliberalism) has resulted in broad ranging systemic fragilities both within and between market Nations, as highlighted by the sars2 pandemic, then all eyes are turning towards the balance between efficiency and resilience.

    Resilience affects efficiency in a number of ways with the resilience concept of redundancy being the most pertinent since redundancy means building slack or reserve in the system in case of adverse events where this reserve capacity can be quickly mobilised to deal with shocks. Another pertinent aspect of resilience is the building of multiple production systems so if one fails, then the others allow production systems to continue. Obviously the resilience of this multiple system is enhanced if each separate system has redundancy.

    In other words, resilience is enhanced by duplication, whether in terms of creating reserve capacity or creating multiple pathway systems with each system incorporating reserve capacity.

    When the resilience approach is mapped on to a national economy then reserve capacity is built up where necessary. When mapped on to the global economy, rather than production processes being built up on just in time, globalised, economy of scale production lines, the production systems are, where possible, built up in their entirety within national economies.

    This means rather than a few dominant global production lines that traverse the globe via reimportation in particular, production lines are recreated within national economies in order to create multiple production lines, each at a smaller scale and each in principle serving national needs.

    In this multiple system approach, reserve capacity in each of the systems allows for extra production when necessary, whether for home markets or foreign markets.

    In essence therefore, building up a global system of national resilience in which a few dominant globalised production systems are replaced with a multiple systems approach that primarily serves national needs is the potential panacea for actual, as opposed to non actual, economic growth.

    Yes capital efficiencies are lost, yes profit margins are smaller, yes duplication will result in an increase in resource use and ecological degradation but one, this approach is better than the coming apocalypse of economic collapse, two, it helps to maintain prosperity whilst the human population is growing, three, it provides protection against shocks, four, reshoring production capacity will create more meaningful and dignified jobs even if pay and profits are reduced due to the loss of efficiency and the likelihood of higher commodity prices due to increased competition over scarce resources. Lastly, if production capacity is reshored (or inshored) at national levels, then this capacity can be tax subsidised if and when necessary through cheap debt or monetary wizardry.

    The goal of course is steady state, so not economic growth for the sake of wealth creation but economic growth for the sake of reconfiguring our systems towards resilience and therefore wealth sufficiency.

    Ideally, this resilience transition with respect to national economic planning and investment needs to be done within a global framework and done in conjunction with the Sustainable Development Goals including the addition of humane human population stabilisation and degrowth in order to limit ecological transgressions.

    Something definitely worth considering unless you or anyone else perceives a fatal flaw.

    • This type of resilience thinking approach seems ‘somewhat’ implemented in the UK defence sector.

      Trying to maintain the ability to ‘build your own’ and ‘rapidly scale it up scale up’ was an important consideration in the Strategic Balance of Investment studies I was working on at UK MoD.

      Many studies demonstrated that procuring or modifying a system produced abroad was far more cost-effective in the short term. But the need to maintain an ‘at home’ or ‘redundant’ manufacturing capability resulted in part, or all, of the programme ultimately being delivered by facilities in the UK, even though it is less ‘optimal’ in the short term.

      So, this type of ‘long term resilience’ thinking and cooperation between stakeholders can be done. I’ve seen it happen. So that’s good I guess.

      However, maybe it can only be done in sectors where a relatively long term thinker (i.e. governments) are the most important customer/investor and can influence the industry to act this way?

      I can’t imagine sectors where the primary customers/investors have short term interests wanting anything over than the most cost-effective outcomes.

      Can only ‘big government’ ultimately produce some sort of resilience in national economies?

      Are free-market practices and acheiving resilience just mutually exclusive?

      *BTW Great article Tim, I was wondering over the past week how COVID could affect ‘the outlook’*

    • Thanks Kevin. Your comments remind me of the 2012 Olympics, when the Army had to step in to take over security after out-sourcing it failed. In military (etc) calculation, reserve capacity is seen as a positive, whereas in JIT capitalism it’s regarded as wasteful. This is another reason why I favour the ‘mixed economy’ of optimal private and public provision. I’m quite sure that either nationalising all plumbers, or privatising the police, would be a disaster.

    • Indeed, there seems to be a trade-off between resilience and efficiency. Back in March Dennis Meadows (of ‘Limits to Growth’ fame) wrote an interesting piece about this:
      https://www.chelseagreen.com/2020/limits-to-growth-covid-epidemic/
      As Meadows points out, over the past century – and not only since the neoliberal turn of the last forty years – “there has been wholesale abandonment of resilient systems in favor of efficient systems – larger scale, less diversity, lower redundancy”. This has been the case, among others, in the field of healthcare. In the US and elsewhere, there have been constant efforts to make healthcare systems more efficient, i.e. leaner and less costly to run. As the Covid pandemic has exposed, this has dramatically reduced the resilience of these systems, which has been severely tested in many countries in the last three months – and could be even more severely tested in case of a massive second wave. It’s interesting to note, by the way, that the German healthcare system seems to have coped much better than others with the pandemic, while it is regularly criticized – both at home and abroad – for being very ‘inefficient’.

    • Thanks, indeed so.

      My focus now is on the numbers, using SEEDS to work through some of the economic consequences, always presupposing, of course, that’s there’s no really serious second wave.

      Were it not, though, I’d be looking at how this has been handled, pretty well in Germany, and in the Far East, less well in Europe.

      Having passed the grim milestone of 40,000 deaths (officially, but other estimates are a lot higher), the UK has now had 60 deaths per 100k of population, second only to Belgium on this measure.

      How has the UK got this so wrong? It seems to me there are a number of components.

      1. Lack of preparedness – seems to have been very weak, lack of reserve capacity as you say.

      2. Wrong decisions. I was dumbfounded when I heard the health secretary tell Parliament that there was no need to close airports, because air travel wasn’t a major route of transmission (so how else, then, did a person-to-person infection spread across the world from China?)

      3. Wrong public reaction. Most people seem to have acted sensibly, but it doesn’t take too large a proportion of the reckless (crowding onto beaches, demonstrating, and in other ways ignoring the safety precautions) to undermine the good work of everyone else.

      4. Wrong messages (yes, the ‘Cummings Affair’, but also media handling of the issue, whilst self-interested lobbying from certain sectors hasn’t helped).

      5. Wrong expectations. It still seems to be assumed that this will be over pretty soon, and will be followed by a ‘v-shaped recovery’, with little thought seemingly being devoted to other possible scenarios.

      Logic suggests to me that a second wave is likeliest in those countries whose handling of the first one has been the weakest. If this happens – though, of course, I hope it doesn’t – then will the UK (in particular) have learned from previous mistakes?

  17. Dr Tim. Thank you for another thought-provoking and informative article. You talk of a collapse in Sterling as being a disaster (my words; apologies if I have distorted your view). But could there be a silver lining in a massive devaluation? The resulting high cost of imports would result in more manufacturing returning on-shore (though this would take time since you can’t create a factory overnight).
    But do you think it is also possible that the current industrial scale money printing (QE) is a long-term plan which is actually DESIGNED to bring about a devaluation, particularly with respect to far-Eastern currencies? Explicitly planned devaluations are never popular as UK Labour PM Harold “the Pound in your pocket will not be devalued” Wilson found out!

    • Thanks.

      As I see it, a Sterling collapse would be existentially bad. The last time this happened (or was on the brink of happening) was 1976, when the IMF provided a strings-attached bailout, which I think was £3.3bn. However you adjust that for inflation (say £60bn?), it wouldn’t even scratch the surface today.

      The UK has very large debts in currencies other than GBP, which would become so huge in GBP-equivalent terms that default would be inescapable. It depends on imports of food, energy and components, all of which would become unaffordable overnight. UK financial assets (a measure of systemic exposure) are 11.2x GDP, which is a level of exposure bigger than Cyprus back in 2008-09. Exports becoming cheaper wouldn’t help if everything collapsed.

      This is a worst-case scenario which all of us hope won’t happen. But it’s a logical implication if a serious “second wave” slams the economy, causing (a) a GDP slump, (b) a fiscal deficit catastrophe, (c) panic capital flight (easy in today’s fluid global markets), and (d) an all-seller, no-buyer FX market in GBP.

      However low the probability of a serious second wave, the scale of potential consequence (to be multiplied by probability to weight risk) is enormous.

      This might usefully be pointed out to those who “must” flock to beaches, and to those MPs and others who say we “must” re-open international travel.

      Today’s markets take no prisoners.

  18. Excellent as always, Dr Tim; thank you. Credit card lending in the UK was -7.8% in April, meaning that debt is being repaid. While this might be a lockdown one-off, I suspect not, and surely deflationary.
    We are still witnessing the utter distortion of financial markets by the use of QE. By keeping interest rates artificially low, QE has caused the price of all other financial assets to rise, even though productivity has barely improved and ‘growth’ remains lacklustre. The result is massive inflation in the price of financial assets – which markets mistakenly perceive as profit. Its only profit if you can realise it by selling the asset.
    As others have said, the less obvious consequences of QE include massive distortions in investment and business behaviours, including companies leveraging up to borrow more to buy back their own stocks, which makes them more likely to go bust and leads to wider income inequality.
    I am a patriot, not a nationalist, and I fear that my country will face economic ruination by the mistaken policies (mortgage holidays, furlough extension, poor state benefit protection, a messed up NHS – I have a relative whose cancer treatment has been ‘paused’ – loss of workers rights, etc.) and that the people cramming the beaches in the exceptional sunshine we have been experiencing will have a terrible shock come the late summer.

    • Thanks Mark.

      If, as I assume, the -7.8% was outstanding balances, people are either repaying debt, taking out less credit, or defaulting (or a combination of all three).

      For those of us who understand the energy economy, financial gimmickry is an exercise in denial. However you look at it, it’s horrendous. From a market perspective, it’s horribly distorting, whilst from a collectivist point of view it’s extremely regressive, handing welfare to those in least need.

  19. To those whose posts expect some coordinated, rational, resilience: history indicates that as rare exceptions to the rule. I posted a link to Ronald Wright’s _A Short History of Progress_. The book is short and lectures are freely available on the CBC link. The exceptions have more to do with geological luck than to cultural intelligence.

    • Thanks Steve. We’re going to need intelligence, rationally applied, to get ourselves through the coronavirus, let alone de-growth and/or the climate challenge.

  20. Finance; Race Relations; Degrowth; Anarchism
    Chris Smaje, a small farmer in Britain, weighs in with a nice essay:
    https://smallfarmfuture.org.uk/2020/06/covid-19-economics-beyond-austerity-and-debt-finance/

    Above, I advocated for a large dose of anarchism…everybody mostly doing what they perceive the need to do, and bearing the consequences…good and bad. One alternative is described early in Chris’ essay: manipulation of money and the terms of trade. For Britain and the US, a crucial question is how long the financial/ terms of trade ‘solution’ can be expected to continue to work. Dr. Morgan gives us reason to suspect that the answer to that is ‘not very long’.

    Smaje points out the change of direction in China…while agreeing that they have not fully implemented that change of direction. He ends with an exhortation to get out your gardening implements and get to work. That’s pretty much what I had in mind in terms of anarchism. Whether Kropotkin’s observations of factories which worked more efficiently when worker managed still holds true, I have no idea. I do believe that the basics of food, water, shelter, and domestic energy are best provided by people with a lot of freedom and financial accountability.

    Of course, the last thing politicians and corporations want to deal with is a bunch of independent peasants who make up their own minds. And too many people believe in reliance on the government to soften any blows. So it won’t be easy.

    Don Stewart

    • This reminds me of ‘enclosures’, a process by which English common land was turned into private farms. It was particularly prevalent in the era of the Napoleonic Wars.

      Those promoting enclosure contended that large farms were more efficient, which perhaps they were.

      But the real agenda was darker. A person with use of common land could fall back on it, if he didn’t like the wages and conditions offered by farmers. Without the commons, he became a wage slave, with nowhere else to go. Without commons, workers couldn’t afford to be (in the contemporary term) “saucy”.

      Quoting from memory:

      “Crime is great in man or woman
      Who steals a goose from off a common
      But of what crime should be accused
      Who steals a common from a goose?”

    • It has always been my understanding (possibly flawed) that the arithmetic showed that the lords could make more money on a given piece of land with sheep producing wool than with peasants producing mostly self-sustenance. So the peasants had to be driven off the land.

      The consequences included the diaspora of English speaking people around the globe, the loss of Britain’s ability to feed itself, free trade to insure cheap food imports, and ultimately the conclusion that all values are financial.

      Careful studies show that Britain can still feed itself…but it has to bring back small farmers…which implies breaking up big estates. Because of the increase in population, it will also be necessary to restrict inefficient food production methods such as feeding grain to cattle. Which implies that the simple possession of lots of money can’t be allowed to become the sole determining factor of what gets consumed.

      Don Stewart

  21. Tad Patzek has updated his Covid charts and predictions in his latest post on LifeItself. His latest Gauussian graph shows Covid burnout in the US by the end of June, but this is based on data before the effects of the recent riots.

    Seconding Steve Kurtz’s remark above, I point out that Patzek’s article contains a good example of how to drive oneself mad with frustration by viewing the shenanigans of government from the perspective of “what could have been done if only government used reason to pursue the common good” All government acts are then seen as “wasted potential.” At such times it is useful to view the matter from Alfred Nock’s viewpoint: if almost never in history has government actually served the common good, maybe just maybe that is not its actual purpose. Maybe what it actually does time and time again is what it IS and what it is FOR. https://mises.org/library/criminality-state

    Patzek writes:
    “As my agricultural journalist friend, Alan Guebert, wrote in an email:

    “This [the financial aid for small, decentralized butchers by the federal government, TWP] can be done for local food because, after all, what do we have to lose? We will have spent upwards of $100 billion on farm programs alone in the last four years to support a food system that’s clearly failing, and spent another $65-75 billion/ year subsidizing SNAP. What would 10% of that money bought had it been invested on long-term, local food systems that are pretty pandemic-proof, help rebuild local economies, and delivered higher quality food to more people?”

    Please put Alan’s comment in the context of the recent $2 trillion dollar economic rescue package that ended up reinvigorating the dying or dead status quo, and going to big corporations with cash on hand. The little virus that could will continue to expose mercilessly these US federal government fallacies. In case you are wondering, my friends, the US has been thoroughly looted over the last 40 years. And I do not mean a few stores that went in smoke during the last week, but an industrial-scale, white collar looting that has gutted out our US of A., just in time for the virus to demonstrate the effects of this epochal looting to all of us.”

    • You’ve got that mostly right, Tagio. Most who have resided here for those 40 years know it, but perhaps lack the detailed knowledge of how it was accomplished. Seems to me that there have always been opportunists who try to game the system in any way possible. Think the railroad barons, the early coal & oil producers, slave traders centuries ago… It’s not unique to our lifetimes.

  22. Tim, I am not familiar with Jurassic Coast beach, but I infer that it is one of those places where people are expected to take their garbage with them, and to “encourage” this and by assuming that this will absolutely occur because that is what they want you to do, the local government provides no or almost no waste cans for depositing litter. Conveniently, this high-eco mindedness cuts down on government expense because no one has to go collect the trash every couple of days. Then, volunteers will have to do it for free. Win-win for the government budget. We have many such places here in the States.

    So the beachgoer has the choice to leave the trash or to carry it with him or her home, rather than depositing in a trash bin when exiting the beach area. I think that the people who left all that trash for others to deal with are irresponsible dolts, but on the other hand it is equally stupid of government to not provide easy disposal options for an area that it knows is a popular spot.

    • I understand that the site is privately owned.

      Given that the visitors had to bring all their stuff to the beach, taking it back to their cars doesnt seen to be excessively difficult

    • As I understand it, the Jurassic Coast is a conservation area of great geological importance as well as natural beauty. As such, they don’t want it cluttered with bins, any more than with cafes, hamburger bars and similar. There are plenty of other places for anyone wanting those. It seems to me that, knowing this, anyone visiting should take their litter home. It’s really an issue of behaving responsibly.

  23. Saw on today’s news that the opening of McDonalds has resulted in enormous traffic queues.
    Contemplating the pollution caused by the gas guzzlers in the lines makes me despair .
    If second Covid wave comes the additional respiratory weaknesses will certainly cause still more chaos in the NHS.
    In the same vane , I note that Formula 1 is restarting; result further pollution plus wasteful energy depletion.
    Clearly none of those in command have any clue of the energy/ecological/economic connection.

  24. Tim, given that you say a sterling crisis might lead to defaults, do you therefore think the UK is too big to fail?

    • As we enter de-growth, very little is too big to fail.

      The UK has GDP of £2.1tn and financial assets exposure of close to £24tn. However we look at it, that’s hazardous. It seems to me to reinforce the case for prudence and caution.

    • I had two thoughts lately:

      1) If the stock market is increasing in nominal value, but the nominal economy is not increasing proportionally, that amounts to redistribution of economic power upwards (to those who own stocks).

      2) Economic simplification always seem to lead to political simplification; and that is, first and foremost, political separatism. In the UK context (since most readers and the author of the blog are from there, if I understand correctly) that would presumably be separatism of Scotland. I read about the trials of Alex Salmond and Craig Murray recently.

    • The aggregate valuation of the stock market (or, for that matter, bond or property markets) is ‘intangible’ or ‘notional’ value, in that we could never turn the whole lot into cash. Equity markets are supported by monetary policy, so have ceased to be properly functioning markets, but are expressions of central bank priorities.

      The devolved administrations in Cardiff, Edinburgh and Belfast seem to be handling the crisis better than it’s being handled in England – which, admittedly, isn’t saying very much.

  25. Hi Tim

    Let me ask the 64 trillion dollar question.
    One of the basic implications for the degrowth future is that the whole FIRE (finance, insurance real estate) sector of the economy can not continue to function with its current business model.

    Is there a business model that works for that sector of the economy in a degrowth environment?

    I am interested because that sector of the US an UK economy is quite large and has enormous political clout and they will most likely be the biggest obstacle to effectively dealing with the degrowth situation. But I really don’t know which way we should try to push/ pull them.

    • Yes, one could create a FIRE model that works in de-growth (and those who front-run this could steal a march on competitors who don’t).

      But it would make the sector very different, more restrained (voluntarily or not), and a lot smaller.

  26. Sober Look at Renewable Electricity Supergrid Prospects in the US
    From Energy Skeptic (Alice Friedemann):
    “Because it is unlikely we can scale up wind and solar due to intermittency and seasonal issues, and so much power is lost across long transmission distances, and the grid so vulnerable to terrorism, it may not make sense to have a national grid anyhow. It would be better to spend the energy required on insulating homes, energy efficient appliances and vehicles, and other efforts to prepare for the decline of fossil fuels and consequently electricity as well.”

    There is one place in the US where wind and solar come together as both efficient and pretty reliable….eastern New Mexico. Very low population density, desert to semi-desert, very little water. Yet we still pursue electric vehicles.

    Don Stewart
    http://energyskeptic.com/author/energyskeptic/

  27. I’m not a fan of Kier Starmer, but he’s got a point about this. The UK really cannot afford to risk a second wave, yet policy seems to have descended into a shambles. It doesn’t help when individuals behave irresponsibly, or when big businesses argue for what’s best for them rather than for the national interest.

    https://www.bbc.co.uk/news/uk-politics-52900206

    • Starmer is politicking. He has no analysis, no ideas or no recommendations of his own. His tactic is targeted criticisms because he has nothing of his own to offer.

      The government’s approach is pluralistic with the option of people staying at home in self imposed isolation if they wish and allowing people to get back to normality if they wish.

      The Liberal Establishment are purposely obfuscating because their sole strategy is to undermine confidence in the government in order to build public support for a much longer transition period with the EU.

      The Cummings Saga was driven by the same mal-intent.

      Simply put, the Liberal Establishment does not want Brexit.

    • As I’ve said here before, I’ve no time for Kier Starmer. Though I was avowedly neutral over the merits of “Brexit”, once the public had decided, it was the duty of Parliament to enact it. Those who claimed to be trying to a stop a “no deal Brexit” were, in reality, trying to stop the whole thing.

      This said, I do sense that the government’s virus strategy is unravelling, coincidentally or not since the ‘Cummings affair’ (which I’m sure reduced their authority).

      Top scientists have warned that they’re moving too fast, and risking a “second wave” and, as you know, I would err on the side of caution given the consequences if a second wave happens. The message seems much too vague as to what people can and cannot do. They should have foreseen the recklessness that’s been witnessed on beaches. If I was put on the spot about details, then I guess I’d have closed beaches or, at least, given local authorities backing if they chose to do so. I don’t understand the obession with re-opening aviation, not least given how much poorer most people are going to be after this.

      Liberalism, as the term is currently understood, is in deep trouble after this. Popular support for the public sector has risen, some corporates’ PR has been dreadful, and economic issues seem sure to rise up voters’ agendas.

    • Thanks. I don’t see aviation as something that can be re-opened in a safe way, and – pre-crisis, and for other reasons – I’d already contended that we were at or near ‘peak travel’. I can’t see all that many people wanting to get into aircraft given the risk of infection, and/or quarantines if there’s a “second wave” whilst they’re abroad. It’s already become a pretty miserable experience, in my opinion, being crammed in like sardines, but maybe that’s personal preference. It may be that, in the future, we fly less often, more expensively, but in more comfort.

      It’s interesting that airlines are cutting their fleets, whilst aircraft manufacturers are reducing capacity and shedding skilled employees. These are not measures that would be taken if a rapid recovery to prior levels of activity were anticipated.

  28. Needless to say, a $5 trillion economic calamity later, the data is in. The stunning juxtaposition of medical facts from the CDC’s owns data prove beyond a shadow of a doubt that Lockdown Nation never, ever should have been embraced by Trump as the policy of the land. To wit:

    There are 104 million citizens under 25 years of age who have suffered just 88 Covid-19 deaths under the CDC’s expansive definitions, which include both “confirmed” and also “presumed” cases. That amounts to just 0.09% of all US fatalities and an infinitesimal mortality rate of 0.8 per million.
    By contrast, there are 5.1 million persons over 64 years living in nursing and other long-term care arrangements, who account for 45% of all WITH Covid deaths. That’s a mortality rate of 8,000 per million.
    In short, the risk of death for long-term care patients has been 10,000X higher than for the nation’s entire pre-school and school age populations.

    David Stockman.
    Former economic advisor to Ronald Reagan

  29. The problem then arises when all the youngsters have herd immunity and the virus spreads to older folk!
    Aside from that there are still uncertainties about the long term side effects.
    Everyone, including experts, are flying blind.

    • Correct, Wally. And Sweden has just responded that its process failed. See excerpt and link:

      Sweden’s Hands-Off Approach to COVID Has Failed Dramatically
      “Clearly, there is potential for improvement in what we have done in Sweden.”

      “There was no strategy at all for the elderly, I now understand,” Annike Linde, Sweden’s chief epidemiologist from 2005 and 2013, told Sweden’s state-owned broadcaster SVT last week. “I do not understand how they can stand and say the level of preparedness was good, when in fact it was lousy.”

      Making matters worse, keeping businesses open didn’t prevent Sweden from experiencing an acute economic crisis. Its GDP fell seven percent, Bloomberg points out — the largest drop since World War II.

      Those numbers are similar to those of other European Union members, many of which have implemented far more drastic shutdown measures.

      https://futurism.com/neoscope/swedish-officials-regret-death-toll

    • My view is that we need to be pragmatic, and ignore lobbying from interested parties.

      Based on some US and UK numbers for standard industrial classifications, I think we could re-open between 80% and 90% of the economy at relatively low levels of risk. There are some sectors where risk is high, precautionary measures are difficult, and the contribution to GDP is comparatively small.

    • There is a high chance this is politicking.

      UnHerd has numerous pieces including interviews that provides a much broader analysis base.

      The Swedish scientist you are referring to suddenly popped up after Neil Fergerson effectively admitted that his modelling was wrong because his model when applied to Sweden predicted huge levels of Swedish deaths which never actually materialised.

    • Sweden is not like UK & US in having highly divergent political parties. They have a largely homogeneous society. The link in the piece I posted (if you read it) is to a Swedish news item which I translated. The chap is the current chief Epidemiologist. The woman quoted later was a previous one. Here are the current chap’s words:

      Sweden should have taken more measures against the corona virus right from the start. This is stated by state epidemiologist Anders Tegnell in an interview with Ekot.
      – Should we encounter the same disease, with exactly what we know about it today, I think we would land in doing midway between what Sweden did and what the rest of the world did.

      Somewhere in between?

      – Yes, I think there is potential for improvement in what we have done in Sweden, quite clearly. And it would be good to know more exactly what to shut down to prevent the spread of infection better.

      Unlike many other countries in Europe, Sweden chose a strategy that would limit the spread of the corona virus, while protecting risk groups. Don’t push it back as much as possible.

      The strategy has allowed us to have a more open society.

      But it has also caused the spread of infection in society, albeit at a lower level and even if the deaths have dropped, nearly 50 people still die in Sweden a day. And a total of more than 4,000 people have died in Sweden after being infected by the new corona virus.

      State epidemiologist Anders Tegnell has constantly argued that Sweden had the most sustainable strategy, but he is now also self-critical. It is among other things the high death tolls that got Anders Tegnell to think about.

      Have too many people died too soon in Sweden so far?

      – Yes absolutely.

      And had it been possible to do something about it?

      – Yes, that is what we might consider in the future if there was any way to prevent it.

      But while he would have liked to use more measures that pushed back the corona virus faster and longer, he is still unsure of what measures Sweden should have introduced. Or if it is just that we should have introduced all the measures at once instead of step by step.

      – Actually, all countries have thrown everything in right away. Sweden is one of the few countries that has worked up a stop more and more. All other countries started with the huge lot at once, and the problem with that is that you don’t really know which of the measures you have taken has the best effect. Maybe we know that now when you start taking action off one by one, and then maybe we get some kind of lesson about what else, besides what we did, you could do without driving the total shutdown.

    • There’s a balance to be struck here (and I don’t envy anyone in government having to do this, especially with huge levels of uncertainty).

      From a purely economic perspective, it’s imperative that we lift lockdowns. But, whilst the probability of a second wave might seem low (though I don’t think we know), the potential costs of a serious second wave would, very probably, be catastrophic.

      Personally, I would lift lockdowns pretty cautiously, and wouldn’t be re-opening international travel any time soon (especially as so many business meetings can be conducted on line).

    • Thanks. I don’t think we know (yet, anyway), and it certainly seems true that Britain and America are handling this very badly indeed. A view that I keep hearing from experts is that (I paraphrase) ‘there will be a second wave, but we don’t know if it’ll be a bad or a minor one’. Second waves are, I’ve heard, pretty normal for this kind of epidemic.

      What I struggle to understand is why precautionary measures aren’t regarded as obvious, given the risk/return equation involved. In the UK, for example, it seems obvious that beaches should be closed (except, perhaps, to local people), and that international travel should be kept in lockdown until this is behind us. For the public, these are modest sacrifices, when set against the costs to everyone if things go wrong. Airlines seem to use a high-utilization, low-cost model which exposes them very badly to the slightest dip in customer numbers.

    • I know that I am going to get shot for saying this, but in my view there will be no ” Second Wave”.
      Most of the people who were going to die of Covid-19 are now already dead. Those of us remaining, have generally, a strong enough immune system to cope with it.
      I think that governments and other authorities have enjoyed wielding the power that they have over us, so much so that they are reluctant to give it up, and they will be happy to keep us locked down in case we have a third or a fourth or a fifth wave of this pandemic.
      The longer this Lockdown goes on for, the more difficult it will be for the economy to “recover”. In one of your recent essays, Dr. Tim you depicted a curve showing how the Recovery was not going to be a “V”, but as a drop followed by a blip up.
      That “blip up” will get smaller as time progresses, as fewer and fewer people have any money left for any kind of discretionary spending.
      I am reading that UK house prices are falling, as this is a foundation stone of the UK economy, it adds to my fear of a falling GBP, which will further exacerbate this country’s manifold problems.

    • You certainly won’t get shot for that by me!

      I agree that the probability of a serious “second wave” seems low. But the scale of the damage if we’re wrong about that justifies some level of caution. So, if an activity isn’t hugely important, and is difficult to operate with reasonable precautions, then I’d keep it shut for longer.

      I’m now using SEEDS to work through the economic implications. Consensus faith in a ‘V’ recovery is fading, and rightly so. Most people are going to come out of this with impaired incomes, reduced savings and higher debts. Most are likely to have become more cautious. This might be the point of “credit exhaustion”, where people refuse to take on yet more debt, however cheap and easy-to-get it is made. There may be brief ‘post lockdown exuberance’, but it’s going to be tough after that.

    • Both Sweden and the UK are high latitude countries, on the eastern edge of the North Atlantic. Their populations are therefore vitamin D deprived. The US and UK (not sure about Sweden) also have high obesity rates.
      The British government has also spent the past fifty years gradually replacing its population with imported folk from, shall we say, ‘sunnier regions’ of planet earth. These darker skinned folk, are even more vitamin D deprived than the natives and as the BBC article makes clear, they have a much higher COVID mortality rate as a result.
      https://www.bbc.co.uk/news/uk-52894225

      Britain’s high mortality rate is likely a result of a combination of factors. I think it would be premature to say that it results purely from poor government handling of the crisis. As a nation, we have a number of weaknesses that have resulted in mortality rate being relatively high compared to other countries.

    • I’m certainly not suggesting the government is wholly to blame for how badly the virus has been handled in Britain, though big mistakes have been made. There are limits to what government can do when (a) a significant minority of the public acts irresponsibly, and (b) some organisations’ lobbying puts their interests above those of the country.

  30. Here is an article I saw today which I think is rather ominous: https://www.theguardian.com/business/2020/jun/04/coronavirus-crisis-collapse-fossil-fuels-demand

    The Guardian are spinning it, yet again, as a herald of the coming green future. I, despite being a person who fully comprehends that global warming is a threat to humanity, think this must be yet another warning sign. If the notional value of their fossil fuel reserves are marked down in this way, surely their share prices will go down as will their ability to obtain cheap credit. Is this just another domino falling on the way to a horrorshow GFC II?

  31. Dress Rehearsal for What? Energy Transition: Suggested reading and listening

    Richard and Janet Heinberg on trying to live a suburban California life on a solar budget. About 15 minutes. Probably nothing that will shock veterans, but worth reading to understand why Richard is anything but starry-eyed. No politician, and certainly Elon Musk, wants to hear this.
    https://www.resilience.org/stories/2020-06-04/if-my-house-were-the-world-the-renewable-energy-transition-via-chickens-and-solar-cookers/

    A long conversation between some veterans of Biosphere 2. The fully enclosed dome built in Arizona decades ago. Only sunlight in and only a little heat escaping. Most useful I think in terms of the starkness of what it takes for advanced scientists to actually make a living, even with modern tools, on a confined piece of land using agriculture and doing their own cooking and recycling everything:
    I suggest that you go to the website first:

    The Healthspan Solution


    Scan over it quickly and you will understand what the Ray and Julieanna duo are up too. Then click on the podcast on the bar at the very top of the page. Sometimes this doesn’t work too well, but persevere and you will find the Science and Saucery list of podcasts. Click on the first one, dated June 2.

    I’ll just note a couple of points:
    *Animals are useful mainly to help with recycling. The idea of vast herds existing solely to provide meat for humans does not work…unless you have vast unpopulated grassy plains at your disposal. Plants are the key to survival. They all lost weight rapidly.
    *There are psychological rewards when doing everything for yourself. It undoubtedly helped that there were several people in the Biosphere to split up the work. They were appalled at the waste when they re-emerged into ‘civilization’. Since 3 of the conversants have a NASA connection, they remark that almost all astronauts come back with a renewed sense of ‘taking care of Mother Earth’.

    If we plan to take a few billion people into the 22nd century, there is a lot to think about here.
    Don Stewart

    • Anyone who listens to the podcast will appreciate this tweet from Nora Bateson:
      What is information within a system that is in the midst of transformation? Everything seems to refer to something that is currently disintegrating. What’s left? Super local, fine detail, personal experience, hand made & home grown.

    • Good link, Don. And this reflects the situation for a US couple, with a 0.25 acre garden. Britain is so overcrowded that relatively few families have more than a back yard. The sort of people that own a garden big enough to grow their own food, are wealthy enough not to need to. An indication of just how completely out of balance human society has grown in many western countries. Any sort of self sufficient ‘Good Life’ is now utterly impossible for most people, at even the most basic level. Two centuries ago, most people living in western countries were farmers. What a different world we live in now.

  32. on the subject of a second wave, well there almost always is a second wave when a new infectious disease is doing the rounds,
    I don’t think it’s right to imagine that all those likely to die from this disease have died in the first wave, when you look at antibody testing around the world it appears that only 15-20% of people in metropolitan areas have antibodies and hence resistance and when you get out into rural areas it’s only 5-10% with antibodies,
    we are still very far from ‘herd immunity’ which requires 70-80% to have antibodies,
    but they next wave may possibly be less dangerous than the first, there are a few hints that the virus may be mutating into a less nasty form,
    this is both potentially good news about the possible severity of the next wave and also darker news because it tends to back up the possibility this virus may have been man made in the first place,
    I found the emergence of the virus in China quite interesting, when it popped up in Iran and was immediately taking out people at the top of their social order I became quite suspicious,
    both China & Iran have been obvious main targets of the American Imperium in the last year or so and have both been subject to a full spectrum of attack, economically, diplomatically, by subversion internally and in the case of the assasination of the Iranian General even physically,
    I don’t think it’s a wild conspiracy to imagine that neo-con crazies within the US Gov & Intel services might try a dirty trick like using a bio weapon,
    people like John Bolton & Mike Pompeo spring to mind, if John McCain was still alive I could imagine him being for such a thing, there are probably dozens of other crazies within those US organisations who share similar outlooks that we are unaware of,
    the virus is suspicious because it seems to have leaped through two different species of animal and onto humans without being spotted in an intermediary form,
    this process should take at least 30 years when it occurs naturally and the progression ought to be observed, especially with the amount of resources being poured into viral research,
    literally 100’s of billions of dollars since 9/11
    interestingly the literature with the PNAC Project for a New American Century, did include bio weapons, genetically tailored bio weapons to target genetic groups, along with pretty much a roadmap for all the chaos in the middle east that has played out over the last 20 years,
    the virus contains some bits of RNA that are really out of place and don’t fit into the current theories of natural progression,
    even if gene editing, a technology developed by DARPA, wasn’t used there are still the more traditional methods of using cell cultures to accelerate mutation and evolution that have been around for many decades,
    a naturally evolved virus tends to be quite stable, one which has had it’s evolution accelerated in the lab is much more likely to be unstable and relatively quickly mutate into a less potent form when released into the general population,
    I found where the virus first cropped up suspicous, I was already making plans to protect myself in late Feb early March,
    two countries, the US & UK seemed to take the threat of the virus rather casually, they were happy to poke fun at China & Iran whilst they suffered pain and uncertainty but at the same time down played the potential threat at home,
    the US & UK are the central players within the ‘five eyes’ anglophone intel services alliance, one expects them to be pretty well informed, to have access to information that never reaches the public,
    for the US & UK to gloat whilst China & Iran suffered whilst also being dismissive of the threat at home and then as this possible bio-weapon attack backfires and hits the west to play a rather schizophrenic game of taking increasingly severe measures to control it whilst also being very dismissive of the seriousness of the situation seems rather telling,

    I suspect we will never know what really happened, we still don’t know the full story behind JFK’s assassination, what really happened on 9/11, the anthrax attacks that helped push through the Patriot Act, there is still much debate about how the US & UK got hoodwinked into th Iraq & Afghanistan wars, the nature of the Arab Spring, Syria, Libya, in the UK the inquiry into New Labour’s entry and participacion in the Global War of Terror wasn’t conclusive, even the Hillsborough enquiry took decades to reach a conclusion, long after any of the central players had left the centre stage,
    I think I’m entitled to be sceptical and suspicious!

    ok, now the protests, civil unrest, rioting and some looting in the US and the spread of those protests to other western nations,
    the media has been pretty quick to present a uniform interpretation of events, for me it’s taken a while to start forming an idea of what really is happening and it is pretty complex and nuanced,
    there definately has been massive looting in the US over the last month or so, after trying to buy off the population with 450 billion USD the elites and oligarchs seem to have had a feeding frenzy and looted 5 trillion from the treasury with the approval of both main political parties!
    I think people at the bottom of the ponzi pyramid are getting pretty hacked off at this stage,
    when they try democracy and attempt to vote in people to make things better they either get someone who talks the talk but doesn’t walk the walk like ‘Hope & Change’ Obama and ‘MAGA’ Trump,
    when they try to vote in someone else like Sanders or here in the UK Corbyn the very political parties themselves make it impossible to choose a candidate that they themselves don’t approve of,
    I rather think Kier Starmer might as well be Tony Blair, Joe Biden is a bumbling joke and I find Mike Bloomberg, unable to decide whether he’s a Democrat or a Republican but confident he’s a billionaire setting out to buy himself the presidency with his vast personal fortune rather insulting,
    nothing was fixed after the 2008 crisis and no one went to jail over it, we’re in the process of the next GFC and still there’s no sign of anything being fixed or anyone being held accountable, in fact the worst miscreants seem to be being rewarded handsomely!

    we also in the US can see a trend for the police having become increasingly militarised since 9/11 whilst the Intel services over reach and their surveillance of the domestic population would make the East German Stasi green with envy,

    the US security state seems to be configured to supress it’s own population now, rather like the occupation forces in the middle east seek to control and suppress the populations there,

    this technique of kneeling on an arrestee’s neck seems to have arisen from an Israeli security forces tactic and there is plenty of evidence that US police has sought and received Israeli training in policing techniques,

    the media tries to portray these protests through a racial lens, I think that’s a misdirection, although the media focusses on the violence, rioting and looting they try to ignore the very large amount of black, white, hispanic, young, middle-aged and older peaceful protesters that have one thing in common, they are either within the ranks of the unemployed, soon to be unemployed, lower middle class or soon to be ejected from the middle class,
    their one common feature seems to be that they’re members of the increasing precariat class,
    as always we are seeing a multitude of old hobby horses being dragged from the closet and ridden to death, racism, white supremacy, outside influence, etc.

    when liberals call racism they’re trying to distract from the endless class war perpetrated by the wealthy against the masses that a lot of the priveliged liberals actually participate in,

    when Obama calls for peaceful protest followed by voting and democratic change he ignores the fact that the New Democratic Party isn’t the slightest bit interested in representing the people, bringing around change or providing any hope, just platitudes,
    the Democratic Party is now just a wing of the political duopoly retaining power exclusively for the oligarchs and elites,

    the batshit crazy white supremacists just want to agitate for a race war and get a kick out of imagining they’re a notch above people of colour, but in truth they still have the boot of the establishment firmly planted on their faces, just as those people of colour do, at the end of the day,
    there’s even been some horseshit claims that Russia is somehow instigating the unrest, amusing to hear when US NGO’s have been causing unrest and subversion across the world, interfereing in other countries affairs to the point that most of these NGO’s have been banned in places like Russia,
    I suppose it’s only fair for some Americans to imagine other countries are doing exactly what their own country has been doing abroad for decades!

    how the Donald imagines he can declare Antifa a terrorist organisation bewilders me, it isn’t an organisation, it’s a concept and social stance with no coherent organisational form other than small local semi organised groups,
    antifa is shorthand for anti fascist, anyone that doesn’t idolise Hitler & Mussolini or Franco in Spain, Pinochet in Chile etc. is basically antifa, it’s a personal political stance or leaning not an organisation,
    the people that turned out in East London to oppose the march of Oswald Moseley and his British Union of Fascists were all antifa, they were British, immigrants, christians, catholics, jews, their common identity was that they were the precariat of their day who didn’t want a fascist boot stamping on their face to compound their problems,
    interestingly the self appointed heirarcy of the British Jewish population at the time were trying to persuade the poor jewish people in the East End to not oppose the BUF march,
    ironically there were enthusiastic Jewish supporters of Mussolini, Italian fascism wasn’t racial, it hated communists but didn’t have the crackpot racial and eugenic ideas of the Nazi’s, Mussolini actually thought the Nazi’s a bit batty!
    most astonishing of all when Mussolini turned away from the left, left the army and started up a newspaper to develop his ideas of fascism he was given quite generous monthly funding from British military intelligence because they hoped it would help keep Italy involved in WW1,
    many of the British aristocracy and oligarchy have flirted with fascism in the 1930’s
    many US industrialists and financiers were pro fascist overtly prior to WW2, they’re somewhat more cautious since!
    there’s even a photo of our current Queen as a child in the palace gardens practicing her Nazi salute, for elites, aristocrats, oligarchs and monarchs fascism is quite seductive, it aligns with their existing interests,
    anyone disadvantaged, excluded, oppressed and in a precarious posiition is apprehensive of fascism and would find more favor with a socialistic approach,
    it’s simply the two dimensions of class war,
    if you want to get a better handle of what’s happening in America and by extension being reflected elsewhere in the world I can suggest listening to Chris Hedges, theological student, ex middle east desk chief of the NYT and on the ground reporter in Sarajevo and throughout the Balkan chaos of the 1990’s,
    Chris Hedges has written several very good books analysing what’s happening in the USA and has been travelling the entire country speaking to ordinary folk to gauge opinion and attitude,
    he was booted from the NYT for honesty, he was recently one of the writers for the supposedly progressive website TruthDig but apparently when they tried to form a writer union there recently they all got fired and TruthDig is currently shut down!
    another person to listen to for an honest interpretation is Professor Cornel West, he is possibly the only remaining publicly accepted intellectual that hasn’t been co-opted, sold out, bought off or silenced and ejected from the mainstream media,
    he spoke very eloquently about the Attica riots some years ago,
    one phrase that struck home was him talking of ‘the niggerisation of poor working white folk’
    he see’s things clearly as a class war being fought and won by the wealthy, poor white folk are just as much in the shit as poor folk of colour, racism is just evoked to try and split the poor into ehtnic groupings and disipate their collective impact,
    white or black you can side with the wealthy and work for their corporations, staff their police forces or tend to their estates and hope you might get carried along with them but at the end of the day you’re just a house slave, white, hispanic or black, barely higher in status than those schmucks outside working in the fields,

    Cornel West implies that all of us in the precariat are considered to be just niggers now by the elites,
    Gilad Atzmon has said similar, saying ‘we are all Palestinians now’

    as far as the precariats representation in politics, be it Labour, Liberal or Conservative, Democrat or Republican all the parties are uniformly Likudnik and we have been reduced to the status of Palestinians, refugees in our own home countries, devoid of political power or representation and ruthlessly suppressed and exploited by an elite class that is incapable of reforming itself.

    so you could argue to some degree it is the whites or the blacks or the hispanics or the christians or the catholics or the jews or the muslims or the communists or the fascists or the nazi’s or any identity group you might wish to gaslight and berate,

    but in reality it’s the haves which want to have it all versus the have nots who don’t want to have less,
    it’s really that simple.

    sorry, that was a long one!
    I just wanted to cover some background to clarify current events,
    some might think I come over as a raving commie but in reality I’m very much in favor of a mixed economy, one hand washing the other, if communism was the thesis and capitalism was the antithesis then we desperately need a synthesis of the two and a return to the centre, the social democracy that worked perfectly well in Europe and the US after WW2 and during the cold war.

    I have very little hope of constructive and peaceful change in the near future because it seems undeniable that the current elites clinging to the levers of power are too stupid, arrogant and unimaginative to willingly reform themselves and their system,

    I’m afraid Tim is right, maybe we just aren’t smart enough to fix what’s wrong.

    • Long, but interesting.

      Based on what I’m working on now (economic stuff), I think the leadership in government, business and finance haven’t a clue.

      Rather than going into numbers (next article, probably), here is the average person after the worst of the crisis:

      – Poorer – income impaired, savings eroded, debt higher, house price (where relevant) probably trending lower.

      – Shocked, not just by the virus but by revelations of financial fragility and general (including employment) insecurity

      – Cautious, not willing to take on yet more debt, however cheap and easy (so there goes the monetary “fix”)

      – Angry – wants greater security, affordable health care, wants ‘the rich’ and ‘big business; to pay more

      – Mistrustful, about everything from health advice to governments, media, corporates…..

      Economically, this means non-essential purchases stay low (after perhaps a brief post-crisis binge). Politically, it’s an open door for someone like Bernie or, next best choice, Mr Trump.

  33. Matt,

    Good rant and overview. The “elites” only ever listen to their own echo chamber, so the fact that we may be on the verge of revolution always comes as a surprise. The stock market is doing so well!

    When my wife and I visited Ireland for the first time we did a bunch of the standard sightseeing and tours and we heard a lot about how the English treated Ireland for oh, say a thousand years. My wife already knew quite a bit of the history but it was new to me. I had the realization at that time that racism was just another ploy and manifestation of, if you will, the colonial or exploitation, master/slave mindset. Once you start with the goal of wanting and needing to squeeze a surplus out of some group and keep them at subsistence levels so that you can be rich, it literally doesn’t matter what rationalization you form to justify your treatment of them. Skin color is a convenient marker to hang the subhuman moniker on, but as the Irish discovered, being white is not necessarily of any help.

    People are capable of seizing on literally any difference and making that the marker and justification for why the marked group “deserves” its lot in life and “needs” the benefits of the master group’s administration. Race and ethnicity are perhaps the most usual and best markers because they are such obvious differences, but literally anything will work: religion (the failure to be Christian being perhaps the most notable marker that the subgroup needs to be “civilized” and administered i.e., plundered), language, mode of dress or absence thereof (think Amazon tribes, Polynesians), or customs – all will serve.

    You can add to your list above Russell Means’ statement following the treatment of Occupy Wall Street that “all Americans are Indians now.”

  34. @ Matt, I agree with your analysis on the socio-geopolitics. I know someone who works as a right-hand skivvy for a multi-millionaire and they said the attitude of that class was they are like a different species, so when they looked at the rest of us it was like looking down from a plane at an altitude when you can just about make out people as ant sized and we were worth just the same.

    No matter how scary things get now, they wont change because they know they can always divide and rule us, if you have the money-printing machinery, you can always pay the couple off % of the population who’re psychopaths to intimidate the rest into compliance, that’s why they call us the sheeple.

    • Chris Hedges went to school with the children of said elites, and what he says about their behavior is profoundly disturbing and scary. Basically a club of all-out narcissistic sociopaths that think they are above the law, and entitled to whatever they want regardless of consequences. So the Harvey Weinstein or Jeff Epstein depravities were not isolated aberrations, but typical abuse of underlings seen as playthings with no regard for the harm and pain imposed.

  35. thanks Taigo, thanks F1,
    thanks Tim,
    it was a bit long wasn’t it and not strictly economic but more alluding to the socio aspects that then can shape economic behaviour,
    I think you sum up very well the socio stances that will direct economic behaviour in your reply,
    keep it coming!

  36. “During the Second World War, night-time blackouts were imposed, to make it harder for enemy bombers to find their targets”
    This reminded me of a comment Gail the Actuary made recently about Peak Coal being reached in Britain at the time of the WWI and similarly in Germany at the time of WWII.
    Maybe all war is preparation for degrowth?

  37. Elites vs sheeple. Whites vs blacks. Israeli’s vs Palestinians. Tutsi’s vs Hutu’s.

    Like i said; brain quantity is no guarantee for succes.

  38. Hi Tim

    Thanks for a very sobering post.

    Like you I’m fearful of a second wave which could well be, as you suggest, even worse than the first. It’s clear that the government are exercised about this possibility too and, although I agree they may not have handled the current wave well, are fully cognisant of the risks of a second and appear to be acting cautiously.

    I was listening to an interview with Lord Sumption earlier this week; always worth listening to, and he indicated that the Cvid 19 pandemic is actually a rather minor affair in the context of some planning the government did a few years ago for this sort of contingency. If he’s right let’s hope that there is a swift and comprehensive review of this wave in order that we can be prepared for a second or different but more lethal pandemic in the future.

  39. This story is rather worrying:

    https://www.bbc.co.uk/news/health-52944037

    The gist of it is that the R number seems to be moving back above 1 in the UK, even though the official line remains that it’s slightly below. This is of particular significance where numbers infected remain high. Given the widespread flouting of guidelines, this seems unsurprising. Are the consequences of a “second wave” – if it should happen – properly understood?

    • There appear to be a lot of people who are not giving any thought at all to the possibility of a second wave or its consequences. They are getting bored with lockdown and want to do something exciting. I wonder how many are in the younger part of the population and don’t think it will affect them anyway.

    • I suspect most are under 60, Trevor. Unfortunately, many will do as they like unless strict controls are being used. I see that there have been large scale protests with violence against UK police. Memorials against soldiers who died in WWII were vandalized. So called anti-fascists have had freedom due to the Nazi’s losing to the allied powers. Their behavior is an abomination. The dumbing down of the US and UK populations along with massive population growth and “multiculturalism” has made some urban societies into tinderboxes.

    • I’m sure you’re right. Unfortunately, those who think ‘it won’t affect me anyway’ are ignoring the fact that their behaviour may affect others.

    • for the UK I don’t think the first wave has ended,
      so I don’t really think there’s going to be a second wave, more an uptick in cases in this initial wave, a second peak, should we call it a ‘V’ shaped recovery!

      I finally managed to buy a few proper masks from my local Quiki-Mart, made in China of course,
      I wore one inside Tesco’s earlier and as I left a guy was grinning and smirking at me,
      there are a few studies coming out about superspreaders, it seems only 20% of infected people are responsible for 80% of the infections,
      basically a superspreader is someone who spends a lot of time indoors and in close proximity with other people and is taking no precautions like distancing, mask wearing or just cutting back on socialising,
      so really the virus is transmitted by the ignorant, stupid and careless,
      fortunately for the virus such people don’t seem to be in short supply!

      I find it bewildering that there still aren’t boxes of x50 masks stacked on a pallett as you enter Tesco’s selling at near cost price,
      I’ve seen video’s of shops abroad giving each customer a simple mask as they enter, they seem to grasp the benefit and be prepared to absorb the cost,
      why is this so unthinkable in the UK?

    • In the US, profit is all. CVS charges $2.00 for one mask. In some Asian countries they sell for 10 cents.

    • Interesting that after a couple of weeks of the hounding a single person for possibly breaking the lockdown, the BBC is very subdued about the mass protests in London. Lots and lots of quotes from the protesters and ambivalent headlines such as “Are you allowed to protest during lockdown” and “Met Police Chief Commissioner said the protests were “unlawful””. Note the extra quotes around “unlawful”, just to distance themselves a little further.

    • Well, that’s the BBC for you – and the authorities seem to have done little or nothing. Then again, they did little or nothing about beach invasions, either.

      One newspaper is reporting this morning that experts are warning of a second wave, and cautioning against any further relaxation of lockdowns – whilst another reports that the PM wants an accelerated loosening to boost the economy. It wouldn’t surprise me if both reports are correct. Few seem to understand what a serious second wave would do to the economy.

      It’ll be interesting to see what FX markets make of this!

  40. Second Wave?
    I usually glance at the Covid-19 statistics on The Automatic Earth. They currently show a new record in daily new cases. So if we are talking about the world as the relevant geography, we are still in Phase I. But as I have pointed out previously, I do not think that the total world, or even a large country, is the relevant geography. Some researchers worked out the number of people in the US in each metropolitan area who are at significant risk from the virus. The worst was Youngstown, Ohio (which had a huge problem with opioids). I forget the lowest metropolitan area, but a number of places with relatively young and healthy people have low numbers. I suggest that in the US, the metropolitan area is the right geography for a gaussian. For example, Arizona is currently experiencing a big rise in cases, and emergency rooms are almost full.

    Washington State (which effectively means Seattle) had a flurry of cases, which has now subsided. If cases start increasing again in Seattle, then I would consider that a second wave. Should Wuhan start to experience a significant increase, then that would be another ‘second wave’.

    In terms of the lockdown restrictions, the first wave would be the situation where people were pretty much free to move around spreading the virus…with restrictions put in too little and too late. The second wave MIGHT result when the lockdown restrictions are eliminated and people revert to their normal gregarious behavior. We might expect the first curve to be a gaussian, and the second curve to be another gaussian.

    If the virus significantly mutates, then that might lead to a third wave. Or it could become endemic. Or maybe the Russian drug is an effective treatment and we stop paying attention.

    On a cheery note, anything thinking that heat would stop the virus in its tracks might take a look at this tweet from Tucson, AZ:
    “I can barely survive in this 110° heat; and, yet COVID is not only surviving, it’s thriving!”
    Another person in Arizona tweeted that ‘a second lockdown is only a few days away’.

    Don Stewart

    • One Other Interesting Tidbit
      The same Automatic Earth post gives some results of investigations in British Columbia. They find that the virus which struck Vancouver did not come from neighboring Washington State in the US. It came from Eastern Canada, most likely from a big dental convention. It’s easy for me to imagine dentists staring down people’s throats all day being exposed to the virus, and then going to a convention in Vancouver and spreading it pretty widely (who know what those Toronto and Montreal dentists do at the end of a long day of technical talk…do they really want Bill Gates tracking them?).

      One point is that if travel restrictions are a solution to preventing the spread, then they have to include domestic airline trips. In New Zealand, for example, should travel from the north island to the south Island be restricted? In Spain, should travel from the mainland to the Balearics be restricted? Albert Bates, whose official home is Tennessee, has owned a house on a barrier island off the Yucatan peninsula in Mexico for a long time. He usually winters there. He got there just before the travel barriers went up, and has been able to avoid orders to ‘get the foreigners out’. Once all the tourists were gone, the locals erected barricades to keep people from Mexico City from crossing the bridge to the island. The point of all this is that travel restrictions are fraught with lots of problems.

      Don Stewart

  41. Dr Tim,
    Thank you for this new post. I see that – in this post and previous ones as well – you use the term “de-growth” to designate “a permanent cessation and reversal of economic growth”, in other words long-term economic contraction. As you know there is a whole “degrowth” movement out there that is gaining popularity and that understands and defines “degrowth” in a different way, i.e. as a voluntary and planned reduction of economic footprint/social metabolism.
    Planned and voluntary degrowth is probably what our societies and our institutions should opt for if, as you say, they/we would have “the systemic intelligence to respond rationally” to the fundamental changes underway. Is this likely to happen? You don’t seem to think so, and I can only agree with you on this.

    I recently wrote about this and framed it that way:
    “As we reach our civilizational turning point, what we would need to do if we would understand our situation is (…) to consciously, voluntarily, decisively and methodically ‘degrow’ our economic footprint to bring it back down to a level that can be sustained by the Earth system we belong to and depend on for our life and survival. We would need to consciously, voluntarily, decisively and methodically ‘decomplexify’ our social, political and technical systems, to make them fit and appropriate for an age of scaling-down expectations.
    Is this going to happen? I may be wrong, but I don’t think so. I mean, some individuals and groups here and there may opt for some sort of ‘voluntary simplicity’ and choose to ‘degrow’ their economic footprint, but this will most likely remain a marginal phenomenon. Overall, I don’t believe that any advanced society will ever choose to degrow voluntarily, nor that it is a choice that could be made and sustained over time in a democratic regime. First, because it would be incompatible with individual freedom as it is understood in the West. Second, because it would inevitably prove detrimental to the material circumstances of a majority of the population, which would exacerbate social tensions and dissensions and most probably lead to a quick reversal of that choice. By the way, in the coming months the COVID-induced economic depression will probably make it clear that most Westerners do not like at all the prospect – and for many the reality – of having to live with less. Third, because competition remains the main organising principle of international relations, meaning that for any society choosing to ‘degrow’ there would always be others that would be most happy to climb up the economic footprint ladder – and grab the power that results from it.
    Similarly, I don’t think that any advanced society will ever choose to decomplexify voluntarily, as the mastery of complexity is precisely what makes them ‘advanced’, and as it is also a fundamental human quest that cannot be tamed universally and indefinitely”.

    The coronavirus and the fate of industrial civilization

    I think there’s little doubt that degrowth, as a political ideology, will continue to attract more and more attention in the coming years, because it is one of the only concepts around that explicitly goes to the heart of the issues we’re facing. Yet for all its merits it will most probably have zero impact on our collective choices. Economic contraction will be our reality and we will have to adapt to it one way or another, but degrowth cannot be and hence will never be a conscious and long-term collective choice.

    • Thanks.

      I’ve always acknowledged the wisdom of the idea that we should choose de-growth, but I’ve never believed it likely that this would happen. I can’t see the public – or, perhaps more to the point – the ‘high command’ in government, business and finance buying the idea.

      My point is a different one. It is that economic prosperity is a function of surplus energy, which is being squeezed ever tighter by the relentless (and exponential) rise in ECoEs. This, by the way, isn’t a forecast, but an observation.

      On this basis, choice doesn’t come into it. De-growth is forced upon us by the energy equation. In the West, this has been the reality for a long time, and our exercises in denial, carried out through financial adventurism, are as hazardous as they are futile.

  42. Pingback: #173. The affordability crisis | Surplus Energy Economics

  43. The barbarians are now running amock. We let these barbarians into our country by the million. I don’t know about anyone else, but I’m not really feeling ‘culturally enriched’ by any of this.
    https://www.dailymail.co.uk/news/article-8396511/Moment-Black-Lives-Matter-protesters-tear-statue-17th-century-slave-trader.html

    I find it amusing and puzzling, that so many people commenting on this blog are critical of economic globalism, and can see its contradictions, yet cannot understand why a globalised, multiracial population is such an unworkable and disastrous idea. They smugly criticize the ruling elites for their economic delusions, whilst clinging to their own delusions.

    The strongest societies are those in which citizens share common ties of shared ethnicity. No sensible person would try and deny this. Why would anyone think it a good idea to deliberately import millions of people that have nothing in common with the native population and then insist they be given the same freedoms and rights as the native population? It is asking for trouble. And it should come as no surprise when these people exploit the weakness of the host population.

  44. Gail Tverberg is 100% in sync with the thinking behind this site in a new article:

    https://www.zerohedge.com/energy/does-increased-violence-reflect-energy-problem

    Her predictions as far as they can be described as that make sense and I can see every kind of bigotry crawling out now to try kick-start resource wars. Both my grandparents fought in WWII and lived to tell me there’s nothing good about war and no real winners. I’ve generally found that that’s the difference between genuine soldiers and bar-room braggarts, they know the difference and don’t even like to talk about it let alone fight for no good reason. Sadly, as always it wont be their decision, they’ll be led by today’s charlatans who’ll impoverish us all with their promises and lies.

    • George Bernard Shaw said that wars were declared by old men, and fought by young ones, and that it would be better the other way round!

    • Though our approaches differ, I’d be the first to say that she’s very good indeed. I’ve not had time to read her latest in detail, but will.

      Humans have a history of fighting to grab ‘our’share of anything going, and this gets intensified in conditions of scarcity. Collectively, this isn’t very rational behaviour, but then we’re not as a rational a species as we’d like to think we are – after all, a rational species wouldn’t have developed nuclear weapons.

      Perhaps it was ever thus. An exam question when I was at university asked us to discuss this quotation – “When Empire [imperalism] comes in at door, intelligence flies out of the window” (Lord Peter Wimsey, quoted from memory, in Gaudy Night, I think).

  45. I have read several of your posts now and the analysis seems spot on but a couple of questions if I may about the basic principles “The first principle is that all forms of economic output – literally all of the goods and services which comprise the ‘real’ economy – are products of energy.” This is clearly true at a basic level and seems especially applicable in the case of industry, but how does the concept of added value fit in especially in relation to services or goods whose value has little to do with the amount of energy used? So for a given service such as hairdressing two stylists could expend the same amount of energy but one’s work could be valued much more highly than the others. Similarly an artist could paint a portrait worth one million – a value that has little relation to the energy input. These values could be ‘re-energised’ if exchanged for items in which the energy input is a signifiant component but what if they are exchanged for other goods and services?

    • These are interesting questions!

      I must start by saying that the enery equation determines our prosperity, but it’s up to us how wisely or unwisely we use it (as stated in the latest article). Second, the role of human physical labour in the modern economy is tiny, my calculations suggesting less than 0.7%, so our role, overwhelmingly, is directing energy, rather than supplying it.

      So let’s say that two artists, and two stylists, use the same amount of energy in performing their respective tasks. In each case, one uses this energy wisely or effectively, and the other doesn’t.

      Moreover, the price of what they produce is subjective – pricing, considered as a whole, determines how we distribute the products of energy between us. The value of a bottle of drinking water to one person living in a wet country, and to another stuck in the desert, could differ very dramatically, even though the bottle of water itself, and the energy that went into supplying it, are the same.

Comments are closed.